Você está na página 1de 98

Musculoskeletal and Immunology (Cont)

Q 46
After running the appropriate tests. you prescribe medications and
schedule follow­up visits for the patient Doe month after the initial
visit. she presents to your office and says that all her symptoms have
resolved She feels 'quite good' but she is concerned about die
prognosis of her disease and the possibility of recurrent joint
problems. Physical examination reveals complete resolution of the
joint swelling. Which of the fallowing is the best response to this
patients concern?
A The disease is self­limited and no long­term sequelae are usually observed
B. The disease may result in joint deformity if not treated appropriately
C. Joint deformity is not characteristic, but renal involvement may be severe
D. Joint deformity is not characteristic. but cardiac involvement may
be severe c'
E. Joint deformity is inevitable in long run
Explanation:
Viral arthritis is typically self­limited and of short duration.
Therapy is generally directed at tile relief V symptoms and
maintenance of function_ therefore, patients are treated with simple
analgesic and anti­inflammatory drugs. Physical and occupational
therapy may be employed if required to maintain or improve function.
(Choice E) Joint deformity is a very uncommon complication in these patients,
(C h o B) Rheumatoid arthritis can cause significant joint deformities
without appropriate long­term treatment.
(Choices C and D) Joint deformities are less characteristic for other
systemic diseases such as SLE and ­he urr atic fever, bJt involvement
of other organs rri ay be significant in these diseases.
Educational Objective:
Viral arthnus is typically self­limited and of short duration.
Q 47
A healthy 43­year­old Mexican man comes to your office and complains
of low back pain. The pain increases in severity when tie lifts
objects and when he stands for prolonged periods of time He has had
these symptoms for the past two months. He was evaluated by another
physician six weeks ago, who recommended bed rest for frve days and
started him art naproxen (to be taken 'as needed' for pain). He also
received three sessions of physical therapy, after which there was an
inih al improvement of his symptoms; however, the pain recurred three
weeks ago, and this prompted him to seek another physician's (your)
medical opinion. He has no other medical problems. He has been in the
United States for the past year and is a constructor worker He denies
the use of tobacco. He drinks 1­2 bottles of beer daily. He takes no
medications. His 'vital signs are within normal limits Examination
shows moderate paraYertebral muscular
tenderness There is no evidence of neurologic compromise The range of
motion of the spine is normal. There is no scoliosis or excessive
kyphosis of the back. Which of the following is the most appropriate
course of action?

A.    Restart physical therapy
B.    Order a magnetic resonance imaging (MRI) oldie spine and reevaluate
C.    Order erythrocyte se cimentation rate (ESR) and reevaluate
D.    Order lumbar spine radiography
E.    Refer the pahent to a rheumatologist

Explanations
According to the American College of Physicians (ACP) guidelines for
the diagnostic evaluation of law back pain, if the patient is younger
than 50 years of age, has no symptoms of systemic disease or cancer,
and has no neurologic a dificites, there exists a  a possibility or
less than one percent that the patients has a condition other  than
musculoskeletal disease. No Further evaluationftesting is necessary in
such patients, however, if the patient does not respond to at least 0
weeks of conservative therapy, an erythrocyte sedimentation rate (ESR)
test is recommended. The ESR is increased in infectious and malignant
conditions A rate higher than 20mmPhr warrants further investigation
with imaging studies.
In this vignette, the patient's history of recent migration from
Mexico makes his case highly suspicious for an underlying infectious
condition g tuberculosis, osteomyelitis, and brucellosis) which is
compromising his steciarn Euler system.
(Choice B) MRI or CT scan of the spine is indicated in individuals
with evidence of possible spinal sten osis, acute radiculopathy with
urinary retention or bilateral neurologic compromise (possible cauda
equine or cord cornpression), or non­complicated radiculopathy that
does nut improve after 4 weeks of conservative therapy
(Choice D) Lumbar spine radiographs are not recommended at this stage,
unless the patient has a history of cancer or abnormal ESR. The
sensitivity of this test is much lower than that of CT or MRI.
(Choice A) The effects of physical therapy in the management of simple
musculoskeletal back pain are not Clear. One third of patients improve
after one week of conservative therapy, and most become asymptomatic
after six weeks with or without physical therapy Restarting physical
therapy in this patient may rot prOvIde arty benefits.
(Choice E) AI this point, mere is no need to refer the patient to a
rheumatologist because there is no evidence of the umatologic
condition. Further evaluation should be done by the primary care
physician.
Educational Objective:
Patients who present imth simple back pain and who have no risk
factors usually have a benign, musculoskeletal etiology, and
successfully respond to one to sixweeks of conservative therapy. Those
Ms present with complicated back pain and bare risk factors for
malignancy or infection. as well as those who have persistent back
pain, can be evaluated using the ESP test. Imaging of the spine is
required for those suspected with spinal stenosis. and for individuals
with acute or persistent findings compatible with rad iculopathy
Q  48

A 55­year­old Caucasian man comes to the physician because of malaise.
asthenia. generalized weakness. and mild fever He has no other medical
problems. He previously worked in a shipyard, where the work invoked
painting and repairing yachts, boats, and big ships. He quit this job
after 20 years. and has been worlong as a truck dnver for the past 10
years. He has smoked one pack of cigarettes daily for 40 years. His
family history is not significant. He does not have any medications.
His temperature is 37.8 C (100 F), blood pressure is 120/70 mmHg,
pulse is 82/min., and respirations are 12/min. Examination shows some
scaly eczema over the knuckles bilaterafiy. The lungs are clear to
auscultation. There are no heart murmurs. The abdomen IS soft,
non­tender, and non­distended. There is a symmetric decrease in
muscular strength: 4/5 in the upper and lower extremities Weakness is
more marked in the proximal muscle groups Reflexes and the other parts
of the neurologic exam are normal. Which of the following conditions
is most likely causing this paftent's disease?
A Malignancy
B. Viral infection
C Bacterial infection
D. Degenerative disease
E Occupational disease
Explanation­.
The patient has I3ottron's sign, an eczematous, erythematous, scaly
lesion that affects the knucldes bilaterally. He else has proximal
muscular weakness. The most probable diagnosis is dermatornyositis, an
autoimmune disease which is usually secondary to a malignancy when it
occurs in adult men. Dermatomyositis in adults is usually related to
solid tumors. such as lung carcinoma. which must be sought in this
patient The patient's heavy smoking history, as well as history of
exposure to industrial paint, highly predisposes him to a malignant
condition.
(Choice C) Viral and bacterial infections could explain the malaise,
asthenia, arid the mild increase in body perature: however. the
muscular weakness and myalgia in viral infections are generalized. and
not proximally distributed
(Choice D) Dermatemyd sins is an autoimmune condition. It is not a
degenerative disease.
(Choice E) The patent's occupational history predisposes him to
accupahonal diseases such as exogenous lipoid pneumonia or asbestosis;
however, these diseases veil not explain the autoimmune condition
manifested by the patient.
Educational Objective:
Dermdtomyosins can he suspected in patents with proximal muscular
weakness and skin lesions. Godron's sign and an
er,ithematous­vialaceous rash in the nose and eyelids (heliotrope
sign) are common findings The latter sign is more common in children.
Derrnatomyositis in adults is usually related to malignancy,
especially if there are risk factors predisposing to malignant
conditions.
Q  49
A 57­year­old postmenopausal female with a history of hypertension.
obesity. and type II diabetes. presents with pain in her right first
metatarsapha lange al (MTP) joint for the past four hours. She is
currently on atenola . hydrochlunothiazide, glyburide and metformin.
Examination reveals a swollen. inflamed. and extremely tender right
first hirITP joint Acute monciarti ruler grow is suspected clinically
Which of tile following statements is true regarding acute gouty
artlintis?

A Allopurinol should be started concomitantry with the other agents
used fortreating acute gouty it
B. Tight blood sugar control is likely la prevent recurrent gouty arthritis.
C. Weight loss and discontinuab on of hydrochLorothiazide is likely to
prevent recurrent episodes of
D. An increase in the serum uric acid level is diagnostic at acute
gouty arthritis.
E Joint aspiration rs contraindicated in acute inflammatory arthritis.
Explanation:
Acute gout t•suuly presents as monoarticular arthritis involving the
first metatarsophalangeal joint. It is common in persons who have
hypertension. obesity. and insulin resistance. Hydrochlorothiazide use
Increases the risk of acute gouty arthritis because the drug can
reduce uric acid secretion into the urine and increase sen011 uric
acid levels. Weight less and discontinuation a hydrochlurothiazide
will most likely reduce insulin resistance and increase the un nary
excretion of uric acid, respectively Reduction of punne­rich foods
(meats and fish) in the diet NA II also cause a significant reduction
of serum unc acid. These activities may help prevent or lessen
recurrent attacks of acute arthritis.

(Choice A) Allopunnol is a xanthine oxidase inhibitor that reduces the
production et uric acid. It is used as a prophylactic agent to prevent
recurrent gouty arthntis. It is never started during an acute episode.

(Cho loe B) Although gout is more common in patients who have insulin
resistance, there is no clear relationship between blood sugar and
uric acid levels Tight blood sugar control has newer been shown in
systemic studies to prevent episodes of recurrent gouty arthritis

(Choloe D) Serum uric acid can be normal during an episode of acute
gouty arthritis. and is not used in diagnosing acute gout
(Choice E) There is no contraindication for joint aspiration in acute
inflammatory arthritis. In fact, the cornerstone for the diagnosis of
acute gout is joint aspiration and demonstration of negative& birefrin
gent needle­shaped crystals

Educational 0 bje cti v a;
Acute gouty arthritis usually presents as monaarticular arthritis of
the first metatarsophalangeal joint. The cornerstone for its diagnosis
is joint aspiration and demonstration of negatively birefringent
needle­shaped crystals Prophryiactic therapy should not be started
during an acute episode Weight loss. a Purine­restricted diet, and the
discontinuation of uricosuric agents (e.g hydrochlorothiazidej may
lead to a decrease in serum uric acid and prevent further attacks.
Q  50
A 61­year­old white male comes to his physician and complains of
severe pain in his right knee for the last 12 hours kris other
complaints are malaise and low­grade fever His other medical problems
include diabetes. hypertension, and chronic renal insufficiency. He
has smoked 20 cigarettes daily for the past AB years He drinks alcohol
excessively He denies any history of trauma He has not been sexually
active these past few days. His pulse rate is Barmin. temperature Is
Tvs C (100.1 F), and blood pressure is 130/87 mmHg. On examination,
his right knee is swollen, warm, arid tender. Complete blood count
shows. hemate c rrt of 40%. WBC count 0111,000/micro­IL: and a
platelet count of 11,000micro­L. I­lis serum creatinine level is 1.9
mgkIL. Blond urea nitrogen is 27mgidL. and serum uric acid is Bmg/dL.
Aspiration of the knee joint is performed ea Compensated, polarized,
light, microscopic analysis of the aspirate show intracellular
needle­shaped, negatively bireMngent crystals Gram staining of the
synuvial fluid does not show any bacteria. What is the most
appropriate next step in the management of this patient?
A Indomathacin
B. Aspirin
C. Colchicine
D Rofecoxib
E. Gricocorticoids
F. Allapurinol
G. Pmbenecid
Explanation:
Thu diagnosis of gout is confirmed by the presence of needle­shaped,
negatively birein fluent urete crystals in the sync daI fluid. To
treat an acute attack of gout, many therapeutic options are available
These include NSAMs, cio lob' ine , and glucocorhcoids. NSAIDs are the
treatrnent of choice for acute gout. Glucocorticuids are used when
NSAIDs are contraindicated for acute gouty arthritis. These Can be
injected directly into the joint or given as systemic therapy. Septic
arthntis may co­exist with gout therefore. gram­staining of the
synovial fluid should be performed before treatment Wit
glucotorticords is started.
(Choice A) Indomethacin is the most commonly used NSAIO in acute gouty
arthritis: however, this patient has impaired renal funchon, which is
a contraindication to die use of NSAIDs Other contraindications are'
active peptic ulcer disease and history of allergic reaction to N
&Alps.
(Chaise m Aspirin should be avoided in an acute attack, as it may
cause hypentricernia.
(Choice C) In elderly patients, colchicine may be quite toxic_
particularly in the presence of renal insufficiency.
(Chia Ice D) COX­2 inhibitors such as rofecoxib can be used to
terminate an attack of acute gouty arthritis in patients Mo are at
increased risk for upper GI bleeding: however. these are also
contraindicated in patterns wit renal failure.
(Cho cot. F and G) During an acute attack, an attempt to reduce uric
acid levels must not be made since this may precipitate an attack of
gouty arthritis. Roth alkopurinol and prohenecid reduce serum uric
acid levels. These agents are used Men acute attacks occur frequently.
but only after an acute attack is over.
Educational  Objective:
Intraartic u lar steroids are the treatment of choice when a patient
with monoarlicular gouty arthritis has contraindications to the use of
NSAIDs or coichicine.
Q 51
A 30­year­old Caucasian male comes to your office and complains cif
pain and numbness in his right wrist and palmar surface of his first
three fingers. These symptoms woke him up last night. He is a baker,
and rolls dough using repetitive movements, including hyperextension
of the wrist arid pronation of the forearm. He remembers having
several similar episodes of finger numbness before, which resolved
completely after a period of rest. His past medical history is
otherwise insignificant. Which of the following is the most likely
localization of the pathologic process in this patient?
A. Median nerve between two heads of pronator  teres
B. Median nerve in the carpal tunnel
C. Ulnar nerve behind the medial ep icondyle
D. Ulnar nerve et the base of the palm
E. Radial nerve at the edge of supinator muscle
Expla n ati o
The clinical scenario described is suggestive of carpal tunnel
syndrome • pain and numbness in the wrist and oalmar surface of the
first three fingers associated with repetitive movements of the wrist.
(Choice A) Median nerve entrapment between the two heads cf pronator
teres is less common, and usually manifests as pain in the velar
forearm. in addition to the symptoms described in this scenario.
(Choices C and D ) Lunar nerve involvement is indicated by tingling
and numbness of the one­and­half fingers an the ulnar aspect of the
hand.
(Choice E) The radial nerve is knovm as the great extensor nerve'
Sensory abnormalities during radial nerve involvement are located on
the dor$al aspect of the forearm and hand.
Eaucational CbJectivel
Pain and numbness in the wrist and pal m ar surface of the first three
fingers associated with repetitive movements of the wrist is typical
far carpal tunnel syndrome
Q  52
A 50  yeari old male comes to the office and complains of muscle
weakness and cramps while climbing the stairs for the past three
months His symptoms are progressively getting worse He denies any
rash, difficulty swallowing. and arthralgias. He has no other medical
problems He recently underwent screening c Dion°  s opy He does not
use tobacco, alcohol or drugs His blood pressure is 120/80  mmHg,
pulse is 72/min, temperature is 36.7C (98F) and respirations are
111min. The physical examination shows bilateral proximal muscle
weakness of both upper and lower extremrties. The cardiovascular and
respiratory examinations are unremarkable. The abdomen is soft. non
tender, and without organomegaly. 'There is no muscle tenderness The
skin examination is unremarkable. His CPK levels are grossly elevated.
Which of the fallowing additional finding is most likely seen in this patient?
A. Increased serum albumin
B. High eosinophil count
C. High neutrophil count
D High serum ferritin
E Normal ESP
Explanation:
Symmetrical proximal­weakness and grossly elevated CPK levels indicate
a primary mus c le disease De rmatornyuskis and polymyas itis are the
main differential diagnoses. Other differentials include mynpathies
induced by drugs. glucocorbcoids. hypotIvoirlism, and malignancy.
serum ferritin levels are acute first phase proteins, and its levels
are likely to increase in any intlarnmatory disorder, therefore, this
Oatien!: with a significant inflammatory muscular disorder is likelyto
have high serum ferritin levels
(Choice E) The patient IS unlikely to have a normal ESR in the phase
Of Significant inflammatory disorder.
(Choices A, B, and C) A high neutrophil count is a non­specific
finding. Although an increased serum eosinophil count is seen with
eosinophilic fas rhs (a rare disorder presenting with muscle pain,
weakness, and myopathic  EMG), other characteristic features Mat
should be present are symmetrical induration of the skin and normal
serum CPK levels.
Educational Objectives:
Proximal muscle weakness with increased CPK levels indicates a muscle
disorder. Inflammatory myosrtis can lead to increased serum ferritin
levels, which is an acute first phase protein.
Q  53
Which of­the following is the most appropriate management?
A Glucocorticoids
B. Hemodialysis
C. ht fluids
D. Mineraiocorlicoids
E Plasmapheresis
Explanation:
majority of patients with inflammatory rnyositis respond to systemic
high­dose giucocartroids (e.g.. predlnisone 1 mgnC.g bodyvireight).
Occasionally, CPK levels decline without any significant improvement
in muscle strength. In such cases. long­berm steroid use is necessary
Other immunosuppressants can he added to treat patients with no
clinical response to steroids, or w create a steroid­sparing effect.
(Cholco D) Mineralocorticaids have na rale in the management at
inflammatory myositis. These are Ono' wily used for patients with
primary adreno c Mica' insufficiency c r type I renal tut ular
acidosis
(Choices B, C, and E)  ALL these therapies are not pnmarily indicated
for patients with inflammatory rnyosrtis.
Educational Objective:
Patents with inflammatory myostis generally respond to high­dose
glococorticoids. Other immunosuppressants are usually added to create
a steroid­sparing effect.
54 Q
A 35­year­old white female with systemic lupus erythematesus was seen
in your clinic. She has been on prednisone for lupus nephnlis lar the
past year. Decreasing the dose of prednisone below 10 mg per day
appears to be associated with an increase in proteinuria. She is
currently on no other medications except for birth control pills. She
denies any symptoms. She has normal menstrual cycles and is sexually
acbve with one partner. She has no children. Because of a fear of
weight gain, she has been restricting dairy products in her diet. She
avoids sunlight due to a fear of skin cancer. She is concerned about
hone lass secondary to prednisone use. Which of the following should
be given to prevent bone loss in this patient?
A. Alendronate
B  Risedronate
C. Fluoride
D. Nasal calrbanin spray
E. Calcium and vitamin D
Explanation:
Glueocorticoid use may lead to a low hone mass clue to multiple
mechanisms Oucocorticoids decrease calcium absorption from die gut,
cause renal calcium wasting, and have a direct anti­anabolic effect an
the hone. Furthermore. they suppress the release of g
onadatrapin­relea sing hormone from the hypothalamus, leading to
central hypogonadism, which can aggravate bone loss.
This patient is on pharmacological doses of glucororticoids, has a
suboptimal intake of calcium and vitamin ID because of poor dietary
habits, and has limited sunlight exposure. She is at an increased risk
for bone loss. Supplementation at calcium and vitamin In is die most
reasonable choice to prevent bone loss. Her current use of birth
control pills is also likely to protect her horn hone loss.
(Choice A and El) Alendronate or risedronete is not indicated at this
time Both of these agents are approved hythe FDA for treanng gl u
°corticoid­in duced osteoporosis. Treatment could be warranted rf bone
mineral density sthdies reveal a 7 score of <­1.5 that continues to
rapidly deteriorate.
(Choice C) Fluoride causes au increase in the bone mineral density of
the vertebral column; however, an abnormal quality of tone and
increased fracture rates have been shown when fluoncle is used to
treat osteoporosis.
(C h °lee D) Calcitonin is a weak anti­resorptive agent that is used
as a nasal spray with a dosage of 200 International units per day,
other dosages do riot seem to work It is not approved for use in
premenopausal patients
Educational CbJeothir e:
Achieving optimum calcium and vitamin D levels is the best initial
step to prevent bone loss due to prolonged giucocorticoid therapy
Q 55
A 43year old Asian­Arneric an male comes For a follow­up visit He has
hypercholesterolemia. for Mich he has been taking atorvastertin 20 mg
daily for the last year. About a week ago, he was seen in the
emergency department (ED) for weakness and intense pain in both of his
thighs following a 10­mile walk sponsored by a local organization. His
serum creannine k]nase (CPK) levels at that time were 500 untts.L
(normal is 25 to 90 units/l. Atorvastatin was discontinued, arid he
was discharged with a prescnption of ibuprofen. He was also atk.is ed
to consult his primary care physician. His symptoms have markedly
improved since his discharge from the ED. His physical examination is
normal. His most current lipid profile performed about fifteen days
ago revealed a total cholesterol level of 150 mg/dL, LDL level of 90
mg/dL, trigYceride level of  100. and  HDL level of 40  He wants your
white regarding restart ng atorvastatin Which d the following is the
hest recommendation for this patient?
A Restart atoivastaiin
B. Recheck serum crealinine kinase levels and restart if CPK level has
decreased significantly.
C. Discontinue atorvastatin and start gerreibrozil.
D. Discontinue­ atorvastatin and start nicotinic acid.
E Discontinue atorvastatin and start ezenmibe.

Explanation:
There is a chance that the increase in creatinine kinase (CPK) levels
in this patient is unrelated to atoneastatin use. lie has been
tolerating atotvastatin without any side effects for the past year.
Furtherrnore. the Patient's serum CPK levels were increased following
a long walk An increase in serum CPK levels occurs with exercise, and
this increase is proportional to the intensity of exercise performed
The most reasonable choice in this patient is to recheck serum c re
atinine kinase levels to determine if they have decreased
In this patient, it maybe prudent to check CPK levels a few weeks
after starting atorvastatin therapy A mild elevation of CPK levels is
common arid is not a contraindication to continue treatment with
atomastatin or other statin medications Although There are no
clear­cut guidelines on 'when to stop a statin medication hollowing an
increase in CPK. levels, majority of physicians discontinue stain
medications when the CPK level is more than 10 times above the upper
limit of normal range The chances for statin mvopathy are higher if
statins are used in combination with a fihric acid derivative.
(Choice­ A) Since this patient previously had high serum CPK levels in
the EC), it is prudent to check whether the levels are decreasing
before considering to restart atorvastatin.
{Choices C, D, and E) since this patient responded to atorvastatin in
an excellent rn annerfor the past year. there is no need to switch to
other lipid­lowering medications unless there is a contraindication
for atorvastatin. Nicotinic acid and ezetimibe are used for
hypercholesterolemia. Nicotinic acid can be used for buth
hypertriglyceridemia and hypercholesterolemia. Gemfibrozil is
primarily used for hype rtnglyc endemi a .
Educational Objective:
Physical acnviry can lead to an elevation in serum CPK levels. Unless
necessary, serum CPK levels should not be measured Fallowing an
intense physical activity
Q 56
A  60 year­old Caucasian female presents to your office with several
months history of a sandy sensan on in her eyes. especially in the
evening, and while watching Nor working in front of the computer Her
past medical history is significant for right knee osteoarthritis. She
occasionally takes naproxen to relieve her knee pain. Her Family
history is significant for a father writ hypertension and a mother who
was diagnosed with breast cancer at the age of BS. She drinks one cup
of decaffeinated coffee in the mom ing. Physical examination reveals
several write patches ion her buccal mucosa. Her recent fasting blood
glucose level was normal. Which of the fallowing queslions is most
appropriate in this case?
A. Have you significantly lost some weight over the last year?
B Do you drink alcohol on a regular basis?
C. Do you wake up at night feeling dry, and then drink some water?
D. Do you have any drffliCUIty falling asleep and waking up early in
the morning?
E. Do you have headaches that are mostly located over your temples?
Explanation:
Patients with Sjegren syndrome may have a variety of symptoms because
many organs may be inyotved. This clinical scenario is highly
suggeshve of since syndrome, which is usually manrfested by dry eyes
(keratoconjunctrvihs sicCa) and dry mouth (xerostom a). The patient
has keratoConjunctivrtis since and Drat candid iasis. Complications of
dry mouth include dental caries. candidiasis (unto 70% patients), and
chronic esophag ills. To reveal xerostomia in the patient's medical
history. some yes/no questions may tie asked, such as 'Do you wake up
at night feeling dry, and then drink some water?" or "Do you
frequently drink water W help you swallow some dry foods?" The other
given questions are less relevant in this case
(Chaise D) Sleep disturbances can be observed due to nocturia, but
early morning awakening is typical for depression.
(Choices E and ilk) Headaches over the temporal area, especially when
accompanied by weight loss, may suggest temporal arterdis.
Educational Objective!
To reveal xerostam ia in patients suspected with Si ogren'sisicca
syndrome. some yes/no questions may be asked, such as. 'Do you wake up
at night feeling dry, and then drink some water?' er "Do you
frequently dnnk water to help you swallow same dry foods?
Q  57
A 60­year­old construction worker presents to your office with a
one­week history of back pain. The pain is brief. severe, and radiates
along the posterior surface of the right igh. It is provoked by
bending forward and straining, and abates an tying dawn. His past
medical history is insignificant He has vied several over­the­counter
pain relievers without success Which at the following is most likely
to he present in this patient?
A. Night pain
B. Morning stiffness
C. Straight­leg raising test positive at ea degree angle
D. Trigger points
E. Neurogenic claudicahon
Explanation:
The characteristics of the hack pain described in this scenario are
typical for nerve root irritation. The pain is frequently brief and
shooting, and radiates along the corresponding nerve distribution. It
can be provoked by forward pending. coughing, or straining. and it
diminishes on lying down. A positive straight­leg raising test at a BD
degree angle or less is characteristic.
(Choice E) Spinal stenosis is responsible for neurogenic claudication.
In contrast to radiculopathy, the pain in spinal stenosis increases
with extension of the spine and decreases with flexion of the spine.
(Choice A) Hight pain is considered an 'alarm' symptom. Night pain may
indicate the presence of malignancy, infection, or vertebral tracture.
(Choice B) Morning stiffness is typical for inflammatory diseases.
(Choke D) Trigger points are characteristic for fibromyalgia.
Educational Cbjectiva:
Careful history and physical examination are helpful in the
differential diagnosis of back pain. Brief and shooting back pain
provoked by bending forward and straining is typical for nerve root
irritation. A positive straight­leg raising test at EU or less is
usually present.
Q 58
A33­year­­old Caucasian female presents to your office with a
six­month history of told intolerance. When the weather is cold, her
third and forth fingers an both hands get numb and turn blue She says,
wear gloves to keep my hands warm, but st does not always work. Her
past medical history is significant for non­ulcer dyspepsia treated
,fflth  Omeprazole. She does not smoke or consume alcohol, and denies
any recreational drug use. She has no know., drug allergies. She works
as a nurse at a local hospital and considers her job rnoderately
stressful. She is sexually active with her husband and uses condoms
for contraception Physical examination findings are within nonrnal
limits.
Which of the following medications is the best initial treatment for
this patient?
A Prazosin
B   Nifedipine
C. Verapamil
D. Aspirin
E Topical nitrogycenn
Explanation:
This patient presents with symptoms suggesirvo of Raynaud phenomena n,
an exaggerated vascular response to cold temperature or emotional
stress. Calcium antagonists are considered the first•line
pharmacological therapy for these patients. but it should be mentioned
that not all calcium antagonists are equally effectrve Most
dihydropyri dine calcium channel bidders (e g , nitedipine and
arnloclipine), as well
as diltiazern, are proven to work well in these patients Many other
vasodilator agents have been hied to treat this co n than.
(Choice C) Verapamil does not seem to be effective or at least equally
effective as calcium antagonists.
(Chaim A) Alpha­ad renergic h lockers such as prazos in are effective;
however, patients eventually become refractory to these agents after
prolonged usage.
(Choler E) Narnglyaerin is usually used as ark adjunct agent to
calcium antagonist therapy in patients with severe Raynaud phenomenon.
It is not recommended as the first­line agent.
(Choice D) Some authors recommend adding aspirin to the treatment
regimen in patients *di severe ischerni a and necrosis to prevent
platelet aggregation; however, aspirin should he used vath caution
because rt can a ctualtyworsen vasospa sro by blocking Me synthesis of
vasodilatory prostaglan d ins.
Educadonal Objective:
Calcium antagonists. typically nitedipine or amlodipine, are
considered the first­line pharmacological therapy for patients vath
Raynaud phenomenon.
Q 59
The patient returns in one month and complains that the treatment
'does not help much' In addition, she has started to experience
periodic joint and muscle pain She says that she has been adherent tb
the medication arid has tried to avoid going out in cold weather as
much as possible Her  husband even gave up smoking because you had
warned her that passive smoking may worsen her condition. Which of Me
following is lie best next step in the management of this patent?
A. Increase the dose of the medication
B. AM a second agent
C. Order arterial Doppler ultra sonography
D. Obtain ANA and RF
E. Reassure and observe
Explanation:
Raynaud phenomenon may be idiopathic (also called Raynaud disease) or
secondary to other conditions such as connective tissue diseases.
vascular lesions. medications. etc. When managing patients with
Raynaud phenomenon. potential precipitating factors such as
medications and environmental factors g . frostbite. vibration) should
first be excluded The presence of symptoms suggestive of systemic
disease (arthralgias and rnyalgias) and resistance to treatment (such
as in this patient) warrant further evaluation; this may include ANA,
RF, CBC, blood chernistry, urinalysis and measurement of complement
levels.
(Choice E) Patients are diagnosed with Oman/ Raynaud phenomenon when
there are no historic clues to a secondary condrtion, normal physical
findings and no ischernic digital lesions. No specialized studies are
indicated in these patients
(Choice C) In patients who have symptoms suggestive of vascular
lesions (e.g., asymmetric involvement and deficient pulses), further
evaluation with digital plethysmograply. arterial Doppler
ulirasonography or angiographyshould he considered.
(Choices A and B) It is not reasonable to change the treatment without
first excluding a systemic disease that may be responsible for the
condition.
Educational Objective:
The presence of symptoms suggestive of systemic disease (arthralgias,
rriyalgias, fever, and weight loss) in patients with Raynaud
phenomenon warrants further evaluation. this may include ANA. RF. CBC,
blood chemistry) urinalysis and measurement of complement levels
Q 60
A 30­year.old African­American female presents to your office 'with a
foul­week history of fahigue, low­grade fever, and bilateral ankle
arthritis Her medical history is significant for periodic headaches,
for which she takes acetaminophen She works as a data analyst at a
healdi department Enlarged cervical lymph nodes are noted an
examination A node biopsy shows non­caseating granulomas. Which of the
following is the most likely additional finding in this patient?
A Bilateral hi'ar adenn pathy
B. Maculopapular rash
C. Elevated serum calcium level
D. Hepatosplenomegaly
E. Facial palsy
Explanation:
This patient presents with signs and syrnpterns suggestive of
saronidosis. Lung involvement is present in up to 90% of patients with
this condition. The typical presentation is bilateral hilar lymphaden
apathy with or without pulmonary parenchymatous changes. Sometimes,
only pulmonary involvement without adenopathyis present.
(Choice  B) Skin involvement is present in 20% of the cases.
Maculopapular rash is the most characteristic lesion.
(Choice  D ) Hepatomegaly andkir splenomegaly are present in 2o­25% of
patients.
(Choice E) The incidence of musculoskeletal and neurologic
complications in patients with sarcnidosis is 5­10%
(Choice C)  Increased renal calcium excretion is present in 50% of
patients and hypercalcemia in up to 20% OF  Cases.
Educadonal Objective:
Lung involvement is present in 90% of patients with saropiclosis. The
classical presentation is bilateral h liar a denopathy.

30 questions
Neurology

If there are images in this attachment, they will not be displayed.
Download the original attachment
Neurology

Q 1

A 62­year.old Caucasian female is brought to your office by her son
because of progressive forgetfulness over the last year Her son says
that she is less active than she used to be. and she has some
difficulty waking she denies any significant weight change, muscular
weakness or urinary incontinence Her past medical history is
significant for exerbonal angina and hypertension. Her current
medications are rnetoprolol, ambdipine and aspirin. She does not smoke
or consume alcohol. Her father had hypertension and stroke, while her
mother had kzheimer disease. She is oriented to time arid space, but
has an impaired short­tern memory and increased reaction time. Her
gait is slightly unstable. instability is more prominent Wien she
closes her Efts Vibratory sensation is decreased, and deep tendon
reflexes on the lower eAremities are hyperactive.

What is the most likely cause of this patients condition?

A Increased C SF pressure

B. Diffuse cortical atrophy

C. Mineral metabolism abnormality got

D. Vitamin deficiency

E Vascular occlusion and tissue necrosis

Explanati o
The two most important reversible causes of dementia that may account
for this patient's condition are normal pressure hydrocephalus and
vitamin B12 deficiency. The latter condition is more likely because of
the patient's physical findings of impatred vibration sensation and
hype n­elleAa These neurologic abnormalities reflect subacute Combined
degeneration of the dorsal (posterior) and lateral spinal columns,
which is relatKiietii specific for vitamin B—12 deficiency.

(Choice l Although normal pressure hydrocephalus manifests as cog
rdive dysfunction and gait disturbanCe, urinary incontinence is
usually present and neurological abnorrnalibes are not typical.

(Choice B) Before considering a diagnosis involving an incurable
dementia such as Alzheimer disease, other possibilities (especially
the reversible causes of mental impairment) should be considered.

(Choice E) Vascular dementia may be Considered; however, the patienr s
particular neurologic findings do nct support this diagnosis.

(Choice C) Although Wilson's disease causes cognitive dysfunction,
this manifests much earlier in life.

Educational Objective:

Reversible causes of cogn dye impairment should be considered in every
patent who presents with dementia.
Q 2

Furthur evaluation shows mild increase in hilirulain. VVhich of the
following veil most likely explain the elevated Inlirubin level in
this patient?

A. Infra vascular hemolysis

B. Liver parenchymal damage

C. Ineffective erythropoiesis

D. Impaired bile excretion

E. Impaired hepatic conjugation

Explanation:

Ineffective erythropotesis is a well­known phenomenon that occurs in
patients with vitamin B12 deficiency. The mechanisms that lead to this
phenomenon include defective DNA synthesis with megakaryohlastic
transformation of bone marrow and intramedullary hemolysis Although
intense erythroid hype ro lasia occurs in The bone marrow. the
erythroid precursors do not mature normally. and subsequently die in
The bone

marrow, causing anemia and indirect hyperbilirubinemia. The markers of
hemolysis (e g , LDH( are usually elevated in such patients.

(C h Old A) Intravascular hemolysis is typicaltv a more severe
hemolytic event that may be associated with non­cross­matched blued
transfusions. eL•posure to chemicals in patients with G6P'D
deficiency, etc.

[Choke B)Livere parenchymal damage is the cause of jaundice in
patients with hepatitis and liver necrosis.

(Choice D) Impaired pile excretion leads to cholestatic  Jaundice.

[Choke E) Impaired hepatic conjugation is observed in patients with
Gilbert's disease. a benign hereditary c ondition.

Educadenal ObJactIve:

Intense erythroid Verp Iasi a occurs in Me bone marrow of patents kth
'Vitamin  B12 deficiency, but their erythroid precursors do not mature
normally. and these subsequently die in the bone marrow. causing
anemia and indirect hyperbilirubinemia.

Q 3

A healthy 32 year­old Puerto Rican man comes to the physician because
of facial weakness. He woke up This morning and noticed some numbness
in his right cheek Whde he was brushing his teeth, he observed that
his mouth was deviated to the left. Two weeks ago, he developed same
painful nudes in both legs. He thought they were ''allergic bumps,'
and did not Come to the physician because the pain decreased in
severity and the lesions improved. He has no other medical problems.
His 'amity history is out significant. He is a ta:a driver. and vvorks
14 to 16 hours daily. He has smoked one pack of cigarettes daily for
the past 15 years. He drinks one bottle of beer every other day, and
smokes marijuana occasionally He is currently not taking any
medications. I­Its vital signs are within normal limits Examination
reveals a right facial droop His nasolabial fold is deviated to the
left Onlythe right frontal creases disappear on grimacing. The liver
is palpated 2 cm under the nght costal border. There are two
indurated, erythematuus, tender nodules and m u thple brown­red,
non­tender, smaller nodules over the extensor surfaces of broth legs.

Which of the following is the most appropriate screening test for this
patient?

A. Computerized tomography (CT) scan of the head

B Magnetic Resonance Imaging (MR1) of the head

C. Chest x­ray

D Purified Protein Derivative (PPD) placement

E. Complete white count (C BC) and serum Chemistry

Explanation;

Sarcordosis is a multisystemic disease of unknown etiology that
usually affects young people. Its prevalence is high in African
Americans, Puerto Ricans_ Irish and Scandinavians. It many involves
the lungs, pen pheral arid mediasbnal lymph nodes, liver, and skin,
although the nervous system, sal riar,, glands, heart, and other
organs may also be involved. The presence of peripheral nerve disease,
skin lesions. cardiac or hepatic

compromise. and other systemic manifestations is highly suspicious far
this condition. Up to 901% of patients have radiographic evidence of
hilar node enlargement. Chest x­ray abnormalities are more cornrnon in
Caucasians and white Hispanics These characteristics make chest x­ray
a good screening test to evaluate the presence of this disease.

The patient has features (peripheral nerve paralysis, erythema
nodosum) and physical findings (probable hepatic compromise)
suggestive of sarcoidosis Erythema nodosum is a lesion characteazed by
painful and erythemato us subcutaneous nodules, which are usually
distributed bilaterally over the lower extremities. A chests­ray
should be performed, and this may reveal bilateral limp hadenopathy or
pulmonary infiltration.

(Choice A) A neurologic disease cannot fully account for the patient's
signs and symptoms. His facial Paralysis is peripheral, and not of
central origin. Brain lesions such as stroke or WI­Cars are not
suggested by such findings For these reasons, CT scan of the head is
not needed.

(Choi carB) Although multiple sclerosis can be suspected. the
patient's other findings (e.g.. possible liver disease and erythema
nodosum) cannot be explained by this diagnosis Hence, MRI is not
required.

(Choice E)  Serum chemistry and C BC are not useful to establish the
diagnosis. These may confirm the existence of a systemic disease, but
will not specify the etiology.

(Choice D)Adthough erythema induratum of maim may be confounded with
erythema nodosum since both are C haracte n zed by nodular lesions in
the lower extremib es, this possibilty unlikely.

Tuberculos is.associated erythema induraturri of Basin is not common.
The characteristic lesions are always chronic and recurrent, tend to
crop around the ankles, and leave a scar or ulceration upon resoNing.
Furthermore, the accompanying disease of die cranial nerves occurs
when the illness is advanced and the patient already has me ningea I
signs. This patient has some nodular lesions that correspond to
erythema nodosum in the resolution phase, and are not characteristic
of erythema induranim of Basin. que to these reasons, PPD placement
will not be as useful as the chestx­ray.

Educational Objective:
Sarcoldosis is a multisysterniC disease of unknown etiology that
usually affects young people. The presence of peripheral naive
disease, erythema nodosum, hepatic compromise_ and other systemic ma
nrfestations is highly suspicious for this condition. tJpto 90% of
patients have radiographic evidence at hilar node enlargement.

Q 4

The patient received a short course of corticosterod therapy for the
facial paralysis, which signrficantly Improved thereafter. Laboratory
tests were ordered during the previous visit. The results are the
following:

CBC

Hb      12.6 g/dL

Ht      38%

MCV      90fl

Platelet count     280.000/cmm

Leukocyte taunt    9,000/cmm

Segmented neutrophils   70%

Lymphocytes     24%

holonorytes     6%

Serum chemistry
Serum Na     138 mEq/L

Serum K     4.0 mEq/L

Chloride     103 mEq/L

Bicarbonate     24 mEq/L

BUN      18 mg/dl

Serum creatinme    0.9 mg/dl

Calcium     11.0 mg/dl L

Blood glucose     84 mg/dl

LFT

Total bilirubin     1.3 mg/dl

Direct bilinubin    0.8 mg/dl

alkaline phasphatase    350  U/L

Aspartate aminotransferase   47 U/L

LFT

Total bilirubin      1.3 mg/dl

Direct biliruloin     0.8 mg/dl

Alkaline phosphatase     350 U/L
Aspartate aminotransfe ra se    47  U/L

Alanine aminoiransferase    50  U/L

The chest x­ray showed mediastinal widening and bilateral interstitial
infiltrates. PP I:1 test was negative.

Which of the following is the most appropriate procedure to confirm
the diagnosis?

A Lung biopsy

B. Skin biopsy

C. Facial nerve biopsy

D  Liver biopsy

E Angiotensin­Converting Enzyme (ACE) levels

Explanation:

The patient's Yaboratory findings suggest the diagnosis of
sarcoidosis_ His blood tests (mildly elevated liver enzymes and
elevated alkaline phosphatase) suggest cholestatic liver compromise
Hypercalcemia is also another common feature The mediastinal widening
is representative of the bilateral hil ar tymphadenop arty. The
diagnosis is confirmed by performing a lung biopsy. ',Mich also
excludes other differential diagnoses

such as infections or malignancies In patients Mb pulmonary
infiltrates (such as in this case), there is an 1:10% possibility that
an en dobronch ial or transbronchial biopsy of the lungs will confirm
the diagnosis.

(Choice D) Liver biopsy is not usually recommended because of the high
morbidity associated with the Procedure Furthermore. nonc aseating
granulomatous lesions cannot be readily distinguished from Other
hepatic granulomatous diseases This modality is therefore only
recommended when other diagnostic procedures have tailed.

(Choice B) Some of the dermatologic manifestations of sarcoidosis are
papules. scar infiltration and subcutaneous nodules Acute subcutaneous
nodules usually correspond to erythema nodosum; however, since
erythema nodosum has multiple etiologies, a skin biopsy will not be so
beneficial in this case.

(Choice C) Performing a nerve biopsy is not recommended unless all
other diagnostic modalities have been tried. due to the technical
difficulties and morbidity associated with the procedure

(Choice E) The measurement of serum ACE levels is only recommended for
screening purposes. It is not useful for confirming the diagnosis
because these values can also be elevated in other medical conditions.

Educational Objective:

Definitive diagnosis of sarcaidosis can he made from the biopsy of the
easily accessible lesions. including:

1. any palpable lymph node

2. subcutaneous nodule except erythema nodosum

3. enlarged parotid

4. lacnrnal gland

If there is no easily accessible lesion, hberoptc bronchascooy with
transbronchial lung biopsy is the procedure of choice Nerve and liver
biopsies are not recommended unless all other approaches fail.
Q 5

A 34­year­old. obese. Caucasian male presents to your office with an
abnormal sensation over his right thigh He says, "It gets numb, and
sometimes I feel burning over mythigh " He denies any back pain,
muscle weakness, ar abnormal sensation below the knee His past medical
history is insignificant. He smokes Mu packs of cigarettes and
consumes 1­2 bottles of beer daily. He is a construction worker. lie
frequently works in a squatting position. Physical examination reveals
an area of decreased sensation over the right a nterolate ral thigh.
There is no muscle weakness. Deep tendon reflexes are symmetrical.

Which of the following is the most likely cause of this patients condition?

A. L5 radiculepathry

B S1 radiculopathy

C. Femoral nerve entrapment

D Lateral Femoral cutaneous nerve entrapment

E. Obturatar nerve entrapment

Explanation:

Meralgia paresthenca is a very common syndrome that is caused by the
entrapment at the lateral femoral cutaneous nerve, which is a small,
purely sensory nerve that is a direct branch of the lumbar plexus. It
can he compressed as rt courses from the lumbar plexus, through the
abdominal cavity, under the inguinal ligament, and into the
subcutaneous tissue of the thigh. Physical examination typicalty
reveals an area ior decreased sensation over the anterolateral thigh
without any muscle weakness or deep tendon reflex abnormalities .

(Choice C) A femoral nerve lesion causes anterior and a nteromedial
thigh paresthesia. but it is uncommon. rt is accompanied by quadriceps
muscle weakness and decreased knee jerk reflex_

(Choices .04 and H) LS and Sl radiculopathies are typically
accompanied by back pain that radiates down the lateral or posterior
aspect of the leg. Weakness of the muscles supplied by the
corresponding segment is usually present.

(Choice E) An obturator nerve lesion is characterized by sensory loss
over the medial diigh and weakness in leg adduction

Educational ObjetIve:

Meralgia paresthelica is a very common syndrome that is caused by the
entrapment of the lateral femoral cutaneous nerve Physical examination
typically reveals an area of decreased sensation over the
anterolateral thigh without any muscle weakness or deep tendon reflex
abnormalities.

Q 6

'Which of the following is the most appropriate next step in the
management of this patient?

A Plain x­rays of the hip and pelvis

B. Electromyography and nerve conduction studies

C Refer to physical therapy

D Prescribe gabapentio

E Advise weight loss and to avoid fight­fitting garments

Explanation:

Meralgia paresthetica is a clinical diagnosis (diagnosed based on the
history and physical findings). The classic patent will present with
complaints of typical pain in the characteristic location, sensory
abnormalities, and absent neurologic abnormalities of The lower leg on
examination Some of the common causes are obesity and the use of tight
garments around the waist Management includes reassurance of the
patient (that this is not a serious condition) and conservative
measures such as weight less (in obese patients) and avoidance of
tight­fitting garments to reduce the pressure on the nerve entrapment
area.

(Choices A and 6) Plain x­rays of the hip and pelvis, electromyography
(EMC), and nerve conduction studies (NCS) are net necessary in a
paaantwrth the oharaotenstic history and physical findings

(Choice C) Physical therapy has no role in the management of meralgi a
parestrietica.

(Choice D) Local corticosterod injection and the use of
arrticonyulstints e., carharnazepine, phenytark, or
g a bapemin) are reserved for patients with persistent symptoms that
are refractory to conservabve measures.

Educational C bj

Merelgia paresthetica is a clinical diagnosis. It is a benign
condition, and treatment includes reassurance, weight loss, and the
avoidance of tight­fitting garments.

Q  7

32­year­old Caucasian man comes to the office for a routine follow­up
visit. He was diagnosed with epilepsy three years ago. and has been
taking carbamazepine 800 mglday ever since. He does not use tobacco,
alcohol or illicit drugs. His family history is not significant.
Physical examination shows no abnormalities The patient is concerned
about the potential side effects of his medication. Which of die
following complications is this patient at greatest risk for?

A Thrombosis

B. Pulmonary fibrosis

C Neutropenia

D. Kidney stories

E Hypernatremia

Explanation:

Carbarnazepine can cause bone marrow suppression. Patients should be
made aware of eady symptoms such as (ever. mouth ulcers. easy bruising
or petechiae, which can be markers of the development of neutropenia,
aplasnc anemia or thrombocytopenia.

(Choice E) Carbamazepine use has been associated with hyponatremia and
SIADH, not hypernatremia. (Choice A) There is a risk of
thrombocytopenia, not thrombosis.

(Choice B) Pulmonary fibrosis or interstitial lung disease has not
been associated with the Luse of this drug.

(Choice D) Urinary Complications include renal failure or unnary
retention, root the development of kidney stones.

Educational 0 bjectiva:

Although the frequer.cy is not clear, neutropenia and bone marrow
suppression due to carbamarepine use are significant adverse reach
ons. Elderly patients are also at risk of SEA.DFI. Because of some
mild amicholinergic effects, there is a risk of glaucoma. urinary
retention or constipation.

Q 8

A 22­year­old Caucasian female comes to the office rah her husband
because of intermittent dizziness. weakness, and an unsteady gait for
the last few days She also complains of unilateral eye pain and loss
of vision. Ocular movements worsen the pain in her eye. Her symptoms
worsen with exercise. Her past history is significant for tingling and
numbness on her nght hand that occurred one year ago and lasted for
three to four days. She has no other medical problems. Her social
history and family history are not significant. She takes no
medication. Her temperature is 37.2 C (99 F), blood pressure is 120/80
mmHg. pulse is 80/rnin, and respired ons are 16(min. On examination.
she has intentional tremors. 3+ deep tendon reflexes. spasticiry, 3/5
power in lower enicrernqies. and positie 9abins4S sign in both the
legs. On eye examination, there is a central visual held defect in her
right eye.

Which of the following is the best test to confirm the diagnosis?

A MRI of brain and spine

B. Lumbar puncture

C. Visual evoked potentials

D. CT scan of the brain

E. EhiG and nerve conduction studies

Explanation;

MS is characterized by d emyel inati on of focal regions in the White
matter of the brain, with a propensity to involve the periventricular
and subpia I rAire matter of the cerebrum, the optic nerves, I] rain
stem. and spinal cord Whenever a female exhibits symptoms in a varied
presentation (i .e interspaced liemeen time periods). multiple
sclerosis must be suspected Optic neuritis that presents as a painful
loss of 'vision is an important presentation of MS Patients usually
have a central visual field defect, and fundoscopy is normal Sensory
ahno rm alio es are other important presentations of multiple
sclerosis. The anset is usually between the third and the fourth
decade, with recurrent attacks of focal neurologic dysfunction
occuning at erratic C and non­predictable intervals. MRI scan is the
most sensitive index of demyelinating lesions.
(Choice B) In patients ',path multiple sclerosis (MS), C SF
examination shows normal pressure, normal total cell count, and normal
total protein concentration. The predominant cell type is the T
lymphocyte. Although the total protein concentration is normal. immuno
globulin levels are high relative to other protein components. The
predominant immunoglobulin type is IgG, I g M and IgA are also
increased. Increase in immunogiobul in G levels relative to other
proteins can be represented as the I g0 index, which is elevated in
9IA of patients with multiple sclerosis They may be found in diseases
other than M115, such as neuropathies, chronic central nervous system
infections, or viral syndromes; therefore, the presence of o ligd c
lona I hands does not confirm a diagnosis of multiple sclerosis due to
its high false positive rate (1356).

(ChOloo C) visual, a uditoiy. and somatosensory evoked response tests
are of value in identifying silent lesions

(Cholera E) Nerve conduction cannot make a diagnosis of MS. but can
aid in locating the deficit. They are useful when evaluating nerve
disorders in the extremities, and may identify lesions that are
clinically silent. Urodynamic studies often aid in the investigation
and management of bladder symptoms.

Educational Objective;

Multiple sclerosis is best diagnosed with MRI. Findings on lumbar
puncture support the diagnosis. Evoked potentials are used to identify
silent lesions.
Q  9

Appropriate investigabon5 confirmed the most likely diagnosis. Which
of the following is the most appropriate noid step in the management
of this patient?

A Start the patient on intravenous m ethylpredni s alone.

B Start the patient on a high, then weaning dose of oral prednis one.

C  Do emergent plasrnapheresis.

D. Start Vie patient on pulse dose cyclophosphamicle.

E. Administer intravenous immunoglohuiins.

Explanation:

Acute exacerbations of MS are treated with IV steroids. Treatment is
indicated when there are disabling symptoms. Mild sensory symptoms
generally do not require steroids Steroid use may hasten recovery in
acute exacerbations. and is the most widely used treatment High­dose
hi methylprednisol one is used for this purpose. Long­term treatment
with steroids provides no benefit and does not prevent future
relapses.

(Chola B) High dose oral steroids with weaning doses have also been
studied in acute exacerbations of MS. and the results are encouraging.
however. oral steroids should not be offered to patients who have
evidence of optic neuritis

(Choice C) Immunosuppressive drugs (e.g.. c­yclophosphamide) may
arrest the course of secondary prooressive MS.
(Choice D) Plasrnapheresis may enhance the beneficial effect of
immunosuppression.

(Choice E) Intravenous i mmunoglo bull ns may reduce the frequency of
acute exacerbations in patients Mil remitting­relapsing MS; however,
there is currently insufficient evidence for this IV immunologlobulins
are therefore not currently recommended for this purpose.

Educational 0 bje ctl v e:

Acute exacerbations of MS are treated with conicosteroids
Beta­interferon or glatirarner acetate is used to decrease the frequen
Ey of exacerbations in patients with relapsing­remitting rig or
secondary progressive form of MS.

Q  10

The patient responded well to the initial treatment, and she became
completely asymptomatic in a few days. She underwent physical therapy
as well. Which of the following is the most appropriate next step in
the management of this patient?

Reassure her arid schedule routine follow­up visit in 8 months.
. Start her on interferon and repeat hilRl in 3 months.
C. Start her on glatiramer and repeat lumbar puncture in 3 months.

D. Start her on very law dose (5 mg) of prednisone daily.

E. Start her on interferon and repeat visual evoked potentials in 3 months.
Explanation:

Gera­interferon or g latramer acetate is used to decrease the
frequency of exacerbations in patients with relapsing­ re m (tang or
secondary, progressive form of multiple sclerosis (M5). Patients
treated with these agents have demonstrated a reduced frequency and
seventy of relapses, a reduction in the development of brain lesions.
and a reduction in future disabilities. All patients with a confirmed
diagnosis of relapsing MS should be started on one of these

A repeat MRI three months after the initial scan is recommended to
monitor the disease actively and progression_ Neurological signs
usually lag behind the MR1 findings.

(Choice D) Long­term treatment with steroids prom des no benefit and
does not prevent future relapses.

Educational Objective:

Patients with relapsing MS should be treated with either interferon or
platiramer acetate. The best way of

monitoring the disease activity is with a repeat MRI scan three months
atter the initial scan.

Q 11

Four months later, she comes to your office for a routine visit and
asks for barite regarding pregnancy,. She regularly takes all her
medications and follows your advice. Which of the fallowing is the
most appropriate course of action?
A Contraception is important while taking this medication.

B. Your disease will get worse if you become pregnant.

C. You can become pregnant safety.

D. Nobody knows the effect of the medication on pregnancy.

E Do not get vaccinated against hepatitis B. as it can exacerbate your disease

Explanation:

In animal studies, both interferon and glahramer were found to hie
teratog enc. arid abortion is a risk Patients who are an these
disease­modifying agents should be counseled regarding effective
contracephon. They should also re advised to stop mese medications
several months before attempting to conceive. If the patient becomes
pregnant *chance, the medication should be stopped immediately. and
she should be referred to an obstetrician for evaluation and
monitoring of fetal development Since Were have been some successful
pregnancies with early exposure to interferon, therapeutic abortion is
not advised

(Choice B) In general, pregnancy has no obvious outcome on multiple
sclerosis (MS). In studies, relapses are reduced up to one­third in
pregnancy, however, relapses were noted during the first six morrhs
postpartum.

(Choice E) Recent studies. including the Nurses Health study. have
demonstrated that patients who are vaccinated against hepatitis B are
no more likely to develop MS than those who are not.

Educational Objactisii•:
Relapses are decreased during pregnancy and increased in postpartum.
Contraception is important if patients are taking disease­modifying
agents. however, therapeutic abortion is not indicated if they become
oreonant with eadv exposure. Vaccination with hepatitis B is not a
risk for MS relapse.

Q  12

Which of the following is associated with the best prognosis in
patients with this disease?

A Optic neuritis as the initial prescnntinn

B. Presence of irrtemuclear ophthalmoplegia at the initial presentation

C. Presence of hemiparesis at die initial presentation

D. Presence of nystagmus at the initial presentation

E Late age onset and male sex

Explanation:

A number of factors are of prognostic value in patients with multiple
sclerosis (MS). Onset at an early age. female sex, and a relapsing
form of disease are associated with a better prognosis.

The initial presentation of MS is also important prognostically.
Patients who inihally present with sensory symptoms or cranial nerve
involvement especially optic neuritis) have a favorable prognosis. On
the other nand. an initial presentation due to involvement of the
cerebellum. trainstem. or pyramidal system signifies a poor prognosis.
Internuclear ophthalmoplegia, nystagmus, and hemiparesis represent
involvement of the I ra instem. cerebellum. and pyramidal system.
respectively, and are bad prognostic features.

Educational Objective;

The initial presentation of multiple sclerosis with optic neuritis or
sensory symptoms carries a good prognosis.

Q  13

A 79­year­old Caucasian man is brought by his Sister to the office.
She thinks that he has been getting increasingly forgetful for the
past 18 months. He can still recall certain things from the past. but
has been having significant difficulty with his short­term memory His
personality has changed recently. and he seems more hostile at times
On direct questioning. the man feels Ste and does not see any reason
for corning to the office. He has a past history of hypertension and
osteoarthritis. His medications are hydrochlorothia ride arid
acetaminophen, as needed. The mini mental status examination done in
the office reveals a mild c=ognitive deficit and a F olste in scare of
MO. The rest of his physical examination is u nremarkable.

What is the best next step in the workup of this patient?

A. CT scan of the brain

B  MRI of the brain

C. Obtain a brain biopsy
D. Obtain a complete blood count

E. Start the patient on donepezil

Explanation:

The constellation of syrnotoms in this case is suggestive of dementia
Dementia of Alzheimer's type is the most common form of dementia in
the elderly population, accounting for about 75 to Elrl percent of the
causes of dementia. Alzheimer's dementia is a progressive neurological
disorder characterized by a gradual decline

in cognitive function, impaired memory (short­term memory in early
stages) and functional impairment, all of which eventually interfere
with the patients daily social and occupational functions. Behavioral
and personality changes are common as the disease progresses. and can
range from passive or withdrawn affectto anger or increased hostility,
paranoid delusions and hallucinations.

Alzheimers disease is a clinical diagnosis There is no single
laboratory test that can confirm or refute the diagnosis; however, it
is important to realize that it is a diagnosis of exclusion. It is
therefore imperative to Rile Out Other psychiatric. neurological,
systemic and metabolic causes of impaired memory or dementia. Some of
the potentially reversible causes of cognitive impairment include
medications (anticholinergics. s edatives­hypnotic s. and p sychotro
pic medications), vitamin B12 deficiency, depression, chronic s
ubdural

ernalorna, normal pressure hydrocephalus, metabolic disorders (e g
hyponatrernia, hypercalcernia, hypothyroidism) and certain infections
(Hf.,, and tertiary syphilis). All patients with suspected Alzheimer's
dementia should be screened for in12 deficiency and hypothyroidism as
an initial part of their workup: therefore, the patient in the above
,ngnette should have a complete blood count to look for evidence of ma
rocytic anemia ( El1 2 deficiency}
(Choices A and E) There is no evidence to recommend initial routine
neuroimagrng with CT scan or MIRI of the brain to make the diagnosis
of Alzheimees disease. It should be considered in younger patients
(less than 130 years) presenting with rapid onset of symptoms or with
associated focal neurological signs.

(Choice C)  AJzheimers disease is a pathologic diagnosis and is
characterized by the eAracellular deposition of amyl c id beta
protein, neurofibrillary tangles and generalized loss of neurons;
however, these findings are generally only detected at autopsy and a
brain biopsy should never be obtained to confirm the diagnosis of

zheimer's dementia.

(Choice E) Donepezil is a cholinesterase inhibitor used for the
initial treatment of patients with mild­to­moderate Alzheimer's
dementia It should be initiated only when other potentially reversible
causes of dementia have been excluded.

EdmitiOnal ObJectlurc

All patient with suspected Alzheimer's dementia should be screened for
vitamin 912 deficiency (nacrocytic anemia with hypersegmented
neutrophils on peripheral blood smear) and hypothyroidism (high TSH
Levels).

Q  14

The appropriate step is taken. A battery of laboratory and additional
testing confirms the diagnosis of Alzheimer's disease. You arrange a
meeting to discuss the diagnosis and available management options with
the patient and his family What is the best next step in the
management of the patient?

A. Admit him to a hospice care

B. Admit him to the geriatric care unit

C. Admit him la the hospital

D Admit him to the nursing home

E. Start him on donepezil at home

Explanation:

The first and most important step in the management of patients with
cognitive Dysfunction is an accurate diagnosis of the type of
dementia. Based on the findings in this vignette, the patient has
mild­to­moderate Atheimers disease. The treatment of Alzheimer's
disease has evoked significantly in the past few years and usually
invokes a multidisciplinary approach. It includes addressing the
safety concerns at home and in the outside environment. prevention 0
falls, treatrnent of behavio ra I disturbances, and preventing the 17
egression of disease with cholinesterase inhibitor and disease modryng
agents

Alzheimer's disease is characterized hy a cholinergic deficiency in
the codex and basal forebrain. causing symptoms of cognitive deficits.
Cholinesterase inhibitors (as the name implies) cause a reversible
inhibition of the enzyme acetylcholinesterase. which is responsible
far the degradation of acetylcholine in the synaptic cleft. This
inhibition leads to an increase in the concentration of acetylcholine
and causes an increase in cholinergic transmission in the synaptic
cleft. The four cholinesterase inhibitors currently a pproyed for
patients with Alzheimer's disease in the United States are tacnne,
donepezil, rive stigmine, and galantamine. Tacnne is rarely used in
clinical practice due to its associated hepatotoxicity. The other
three agents have similar efficacy, and their use is largely
determined by the in cliyidual patient's tolerance and physicians'
preference for a certain drug over another, based on their past
experiences.

Donepezil is used for the symptomatic treatrnent of patients with
mild­to­moderate dementia of Alzheimer's type. In the studies Eompanng
done p ezil to placebo in patents with mild­to­moderate Alzheimer's
disease. there was a small but significant improvement in the
cognitive function in the treatment group as compared to me placebo.
Irtiproyemeqt in o ognibve function and activities of daily Irvin gs
are small, and these drugs usually do not improve long­term outcome.
Du hepezil is administered as a once daily drug. The symptomatic
improvement in patients takes same time to appear, and We patent's
response should be assessed after at least eight weeks of continuous
therapy Treatment should be continued only if the improvement is noted
by the patient's family or on cognitive testing In patients with
progressive symptoms or moderate­to­advanced demenba, a
disease­modifying drug iftilemanhne) should be added to the
cholinesterase inhibitor. Memanbne is an N­methyl­1D­ aspattate INNEN
receptor antagonist, and it reduces the functional decline in patients
,pnth Alzheimer's disease by blocking the action of glutamate on the
%IDA receptors.

(Choler C) There is no indication for hospital admission in patients
with mild dementia. It may be required in certain patients with
uncontrolled paranoid delusions, hallucination or aggressive behavior.

(Choices 13 and DI It is not imperative that the patient be placed in
a nursing home or geriatric care unit at this Lime All family members
or care givers should he counseled regarding the options of long tern
care or institutional care in the form of assisted hone or nursing
home placement for pahents with islzheimer's disease. They should be
educated regarding the progressive nature of the disease and informed
that the needs of the patients usually grow over time, and care giving
at home may not be practical after a certain stage; however. the final
decision on placing the patient in a long­term care facility should be
made by the caregiver or patient's health care proxy

(Choice A) Hospice care is generally considered for patients with a
terminal illness and life expectancy of less man six months It should
not be offered to patients with mild­to­moderate dementia.

Educational 0 bj• cti a a:

Donepe ail or other cholinesterase inhibitors is used for the
symptomatic treatment of patents with mild­to­moderate dementia of
Alzheimer's type. In patients with progressive symptoms or

moderate­to­advanced dementia, a disease­modifying drug (Memantine)
should be added. The final decision on placement of patients in a long
term care facility or nursing home should be made try We patients'
care givers or health care prosy, with appropriate counseling from the
physician.

Q 15

The patient's sister wishes to keep him at home. Even after explaining
the risks and benefits of keeping him at home versus other available
ophons, she remains firm with her decision. What is the most
appropriate response to this situation?

A She is at an increased risk al anxiety, and depression

B. His disease will progress rapidly if he is at home

C C. She will not he able to take care of him at home

D. She can face legal consequences for not choosing the hest therapy

E. You are not adhenng to the standard cif care

Explanation:
The patient has symptoms tonsistentwith the diagnosis of mild
dementia. His disease has slowly progressed over the oast 10 months.
however. it has not reached a point where home care cannot be provided
for the patient The physician's role in such cases is to provide
information and counseling regarding the various available options and
to help the family (sister in this case) make an informed decision His
sister should be cautioned that care giving is generally easy in the
early stages, but grows increasingly difficult and tedious as the
disease progresses overture Certain behavioral and personality changes
in these pabents (wandering aimlessly. repebtrire questioning, social
inappropriateness, a gitab on, aggression. paranoid delusions and
hallucinations) can be quite distressing to the caregivers. As the
disease progresses, the patients become less able to care for
themselves. Mich can impose an enormous burden and mental stress on
the caregivers Caregivers should therefore be cautioned about the
risks of significant anxiety, depression or other psychiatric
illnesses, and they should be advised regularry to take care of their

own needs and medical problems, and provided with resources to arrange
for respite care.

(Choice B) The progression of Alzheimer's disease is not affected by
the place inhere the care is pros vide:I far the patient.

(Ch 0 i C) Whether his sister would be able to provide care far the
patient at home largety depends on her physical and emotional
condition, and on the support that she has available to help her.
Assuming that she would not be able to care for her brother is not
appropriate.

(Choices D and E) There are no standard guidelines on when patients
with Alzheimer's dementia should be placed in a long­term care
facility or specialized nursing care home. The decision is based on
multiple factors including the stage of the illness, the patient's
comorbid conditions, and the ability and willingness of the family or
caregivers to provide the necessary care.
Educational Objective

The caregivers (or the family members) of patients with Alzheimer's
disease are at increased risk for anxiety. depression and other
psychiatric illnesses

16

A 67­year­old Caucasian man presents to a family practice clinic for
his Mist visit after recently momng to the area He complains of
frequent sudden severe pains in his ffdrerrirties and a worsening of
his vision Both conditions have been present for some brae, but the
patient admits he 'avoids the doctor as much as possible.' Past
medical history is significant for diabetes mellitus, hypertension.
chronic hepatitis C infection, and osteoarthribs. Current medications
include glyburide, enal alfa interferon, anti eel esoxib. The patient
has a history of traveling widely as a retired officer in the
military, and his social history is remarkable for remote alcohol
addiction and a 25­pack­year history of tobacco usage. He has never
been married but has a history of marry sexual partners Physical
examination is remarkable for EffiAl of 32 I His pupils are noted to
be small and unequal in size. Bilaterally, the light reflex is absent
and the accommodation reflex is prompt. I:lentil:ion is poor. Heart
sounds are normal, and the chest is clear to auscultation. Abdomen is
obese and nontender. Peripherally, reflexes are diminished and
position and vibration senses are absent. Romberg sign is positive.
His most recent laboratory evaluation includes HbAl c =9.3 and
positive rapid plasma rea gin. The most appropriate next step is to
order which of the Following?

A. Serum glucose

B Urine microalburnin

C. Hepatitis B panel
DL vLIIRL.

E. FTA­­ABS

Explanation:

Although now rather rare. tabes dorsalis was previously the most
common manifestation of neurosyphilis It occurs 18­25 years after the
initial untreated infection and affects the posterior columns of the
spinal cord and of the dorsal roots. Abnormal gait, p a re sthesias,
and lightning pains of the blink or extremities are common complaints.
Physical exam may reveal diminished peripheral reflexes and signs of
abnormal posterior column Function, as well as optic atrophy or
Argyll­ Robertson pupils. Since the Argyll­Robertson pupils are also
associated with diabetes mellitus, it is important to confirm the
positme RPR result in this case with a follow­up treponernal test such
as FTA­ABS (Choice E) before assuming the patient actual& has
neurosyphilis Generally speaking. the inexpensive nentrep cinema!
tests SP Ch as VCIRL and RPR measure IgG and 101 antibodies and are
excellent for screening purposes. Positive results kth such tests
should always he confirmed with a trepone m al test such as FTA­ABS or
MRA­TP, as they check for antibodies directed against treponemal
cellular Compone7ts.

Evaluating the Patient's serum gluco e (Choice A) and urine
microalbdrin in (Choice

uld indeed itruvide  some information about his diabetes mellitus
However, the HbAl c result already offers the physician with
substantial insight into the patients recent glucose control. It is
now of more importance to further evaluate the patient for a
previously undiagnosed syphilibc infection.

Hepatitis 6 panel (Choice C) can be performed if warranted by
furtheranefygis of the patients history, but proper exploration of a
Passible syphilitic infection is more pressing at this lime.
Like RPR. VORL (Choice D) is a nontreponemal test susceptible to false
positives in patients with intecbons, chronic liver disease, advanced
malignancy, connective tissue disease, myeloma, histories of injection
drug use, or histories of mule ple transfusions. Older patients are
also at greater risk tor false positive 'with the nontreponemal tests
Because this patient is older and is chronically infected with
Hepatitis C, his posrtree RPR should be followed up with a treponemal
test such as FTA­ABS, not another nontreponemal test.

Educational Objective

The Argyll­Robertson pupil can be symptomatic of diabetes mellitus or
neu ro syphilis. Syphilis can be screened for with a nonrreponemal
test (e.g., RPR) and positive results confirmed with a treponernal
test (e.g., FTA­ABS, MHA­TP).

Q 17

A 21­year­old Caucasian male presents to the office and complains of
periodic severe headaches. His headaches have been present ter the
last several years_ are quite severe, and significantly affect his
quality of life. He cannot provide detailed information about the
headaches, hut menhons that they are one­sided and happen
periodically. He takes acetaminophen to cope, but he is 'tired of
these headaches' and waots to be diagnosed and treated appropriately.
I­lis past medical history is otherwise insignificant. He does net
smoke or consume alcohol. He has no known allergies. His blood
pressure is 120/70 mmHg and heart rate is liernin, The physical
examination is within normal limits
What is the hest next step in the management of this patent?

A Lumbar puncture

B. MRI of the head

C. Headache diary far a week

D. Ergotamine

E Reassurance and NBA] Ds

Explanation:

Headaches are common complaints in clinical practice; hence, knowledge
of the basic management principles of headaches is important. The
primary goal of initial assessment of a patient who presents with
headaches is to assess the probability of a 'secondary' phenomenon due
to mass lesions, infecin on or hemorrhage. The time profile. pattern.
accompanying symptoms (e.g., fever). and physical findings (e.g..
abnormal neurologic findings) will help rule in or out these
diagnoses.

In this case, the patient's history and physical findings (chronic,
severe. one­sided headaches with a stable pattern and normal
neurologic findings) rite out the diagnoses of mass iesion, infection
and hemorrhage; however, the patient did not give enough details
regarding his headaches in order to reach a specific diagnosis. To
correctly diagnose and treat this patient, the characteristics of his
headaches must be clarified first The patient should be instructed to
make a headache diary for one vveek, and this should include the
headaches' frequency. duration, intensity. associated symptoms and
medications used

(choices A and El) Lumbar puncture and hiRl are performed when
meningitis, mass lesions, or hemorrhage are being considered as the
cause of the headaches. in the absence of findings or details in the
history which sill suppnrr such diagnoses, Performing these tests are
not necessary

Educational Objective;

To correctly diagnose and treat patients with chronic headaches, the
characteristics of the headaches must be clarified. Such patients
should be instructed m make a headache diary for one week, and this
should include the headaches' frequency, duration, intensity,
associated symptoms and medications used

Q 18

The appropn ate step is taken. The patient comes back three days later
and complains of lac omation of the left eye and pain behind his left
eye. His headaches were present "on and off" almost everyday, and each
episode lasted for more than an hour. His left conjunctiva is
injected. What is the most likely diagnosis?

A. Temporal arterbs

B. Common migraine

C. Cluster headache

D. Tension headache

E. Allergic conjunctivitis

Explanation:
Cluster headaches represent a distinct type of headaches. True to its name. the

headaches characteristically occur in clusters (groups) The episodes
typically occur one to three times daily Each episode lasts from 30
min to three hours. and presents with severe retro orbital pain.
lacrimation, conjunctival injection, rhino rrhe a. sweating and
pallor. These symptoms usually last four to eight

weeks. Sometimes, no periodic pattern is present, and the episodes
happen without long­lasting remissions (se­called chronic cluster
headaches). Cluster headaches can present as Homer's syndrome.

(Choice 13) Common migraine also presents with unilateral headaches,
but no associated eye and nasal symptoms are present.

(Choice D) Tension headaches are usually band­like, and have none of
The associated symptoms described in this case.

(Choice A) Temporal artentis is not characteristic for this age group.
It happens in older individuals. (Choice E) Allergic conjunctivrhs is
not accompanied by pain and is typically bilateral.

Educational CbJectIve:

Cluster headaches occur in clusters. Characteristic symptoms are
severe retroorbital pain. lacrimation, conjunctival injection, rhino
rrhe a, sweating and pallor. Cluster headaches can present as Homer's
syndrome.

Q  19
The patient further states that although his headaches have been more
severe recently, these have been present for several years without
clear­cut remissions. What is the test chronic therapy for this
pabent?

A. Sumatriptan

B Prednisone
C.Lithium

D. Propranolol

E. NSaJDs

Explanation:

Lithium is a good option for chronic therapy of cluster headaches.
Direct comparison to calcium channel ­antagonists (++erapamil) shOuved
that both agents are equally effective, although verapamil has a
better side effect profile. Ai the same time, lithium has been
demonstrated to be highly effective in Chronic

(nor ­remitting) forms of cluster headaches. This patient seems M have
the non­remitting form because he does not report clear­cut remiss ion
episodes characteristic for classic cluster headaches, therefore,
lithium therapy with careful monitoring of the blood lithium level is
the hest choice for this patient.

(Choice 13) Prednisone is also highly effective. but long­term
prednisone treatment is not recommended due to its side effects.

(Choice A) Sumatriptan is used to cope with acute episodes of vain
(Choice D) Propranolol is an effective agent for migraine prophylaxis.
but is not widely accepted for cluster ',eat/acne prevention.

(Choice E) NS.4IDs are not a reasonable choice for chronic preventive
therapy in this pabent.

Educational Objective

Lithium is highly effective fur the chronic (non­remitting) form of
cluster headaches. Direct comparison to calcium channel antagonists
(verapamil) showed that both are equally effective, but verapamil has
a better side effect profile.

Q  20

Later that day. die patient presents to the emergency department with
increasing retroorbital pain. lacrimation, rhinorrhea and vomiting.
What is the best initial management for this patient at this point?

A. Give IV verapamil

B. Proceed with temporal artery biopsy

C. Order CT scan

D. Give 100%  oxygen via mask

E Institute predoisone therapy

Explanation:
This is a typical 1JSMLE question on cluster headaches. The acute
management of duster headaches usually begins with 10096 oxygen
inhalation, which is surprisingly very effective in the majority of
patients. According to one study, oxygen preAded relief to 75% of
patients with duster headaches. Another option for abortive therapy of
cluster headaches is surnatoptan, either subcutaneous or intranasal
Ergot preparations arid NI5AlDs have also been reported t be effective

(Cho i C9 A) Verapamil is a good preventive agent in patients earth
cluster headaches.

(Choices E and H) Early prednis one therapy and temporal artery biopsy
are indicated in elderly patients mth suspec Coo temporal artenhs

(Choice C) CT scan is warranted if a mass lesion or hemorrhage is
being considered as the cause of the headaches

Educational Objective :

The acute management of cluster headaches usually begins with 10046
onion inhalation. which is surprisingly very effective in the majority
of patients.

Q 21

A 22­year­old white female is brought to your office by her mother
because of recurrent syncopal episodes The first episode occurred
approximately one year ago, when her roommate committed suicide
Several similar episodes then followed, and these were usually
provoked by strong emotions. The episodes are preceded by
lightheadedness. weakness, and blurred vision, and last for
approximately three minutes nth rapid recovery of consciousness. Fier
past medical history is insignificant. She is not taking any
medications arid denies drug abuse. Her blood pressure is 11 M0 mmIllg
while supine and 108(70 reirTiH.g standing Her physical findings are
within normal limits. EKG perforrned one month ago was norrnal. Which
of the following is the next best step in the management of this
patient?

A Echocardiography

B. 24­hour (Holler) monitoring

C. Electroencephalogram

D. Invasive elechrophysiologic testing

E. Upright till table  testing

Explanation

The most probable cause of this patient's episodes is vasovagal
syncope (common faire}_ This is also known as neurally mediated or
neurocardiogenic syncope The clinical scenario described is very
typical for this condition. which includes presyncopal prodrome
(lightheadedness, weakness. and blurred vision), Provocation by an
emotional situation, and rapid recovery of consciousness Vasova gal
syncope is frequently recurrent In this case, upnght tilt table tesung
with or without pharmacologic provocation (isoproterenolli may he
indicated tc confirm the diagnosis.

(Choices E and Ef) 24­hour monitoring or invasive electrophysiologic
testing are indicated when arrhythmia is suspected as the Cd11se of
syncope This IS Usually the case in patients wiith coronary artery
disease and underlying cardiac disease.
(Choice A) Echocardiography can confirm the diagnosis of hyperOophic
cardiomyopattry or acquired valvular defects Generally, if the history
and physical examination are not indicative of a cardiac disease. the
pro habil thy of cardio genic syncope is low

(Choice C) EEG is used when seizure is the suspected cause of syncope,
however, the scenario described (pre syncopal prodrome and rapid
recovery of consciousness) is not characteristic for seizure.

Educational C cti cr.

Vasovagal syncope is the most common cause of syncope. r is frequently
recurrent. Upright tilt table testing may be indicated to confirm tile
diagnosis if the syncope is recurrent.

Q  22

A 22­year­old Caucasian female is brought to your office by her
boyfriend after he found her unconscious on the bed. He says that he
is unaware of her past medical history. but remembers one similar
episode that res never' completely Her temperature is 36 7C(989, blood
pressure is 130/80 mm Hg, pulse is eflimin, and respirations are The
patent is unresponsive te verbal and tactile stimuli Physical
examination reveals a symmetric decrease in muscular tone and normal
deep tendon reflexes. Pupils are symmetric, 5mm, and reactrwe to
light. Imgation of the nght eternal auchtory canal with cold water
reveals a transient, conjugate. slam deviation of gaze to the right.
followed by saccadic correction to the midline. Which of the fallowing
is the most likely cause of this patient's condition?
A. Massive stroke

B Botulism

C. Guillain­Barre syndrome

D. Heroin overdose

E. Psychogenic coma

Explanation;

The clinical scenario described above illustrates the importance of
oculovestibular testing in the differential diagnosis of coma. Caloric
stimulation of the vestibular apparatus is performed by irrigation of
the external auditory canal with cold water. A normal response is
characterized by a transient, conjugate, slow deviation of gaze to the
side of the stimulus (brainstem­mediated), followed by saccadic
correction to the midline (cortical correction). A caloric response
cannot be voluntarily suppressed. therefore. the normal
oculovestroular reaction strong& suggests psychogenic coma.

(C hoic es ); and ID) Metabolic or supratentonaliinfratentonal causes
of coma usually disrupt brainstem­mediated or cortical components of
the caloric response.

(Choices H and C) Botulism and Guillain­Barre syndrome lead to the
paralysis of skeletal muscles. but rarely result in a true coma

Educational Objective:

A iarmalreaction to caloric stimulation of the external auditory canal
swingy ­suggests psychogenic coma.
Q  23

A 27­year­old male presents with progressively worsening headaches for
the past Three months. For the past two weeks, he has felt that he is
losing his balance He denies any motor weakness or sensory symptoms He
does not have any history suggestive of cranial nerve paralysis On
physical examination, his pupils do not reaEt to light bilaterally He
has an impaired upward gaze and loss of optokinetic nystag m us. The
rest of his eye movements are within normal limits. On tandem walking,
he tends to fall on bath sides. The rest of the physical examination
is unremarkable. What is the most likely diagnosis of this patient?

A. Cranlopharyngiama

B  Frontal lobe tumor

C. Pituitary adenoma

D Pineal tumor

E. Multiple sclerosis

Explanation:

A tumor of the pineal gland C haracterisbc ally causes P annaud's
syndrome, which is characterized by the loss of pupollary reachon.
vertical gaze pa ra hysis. the loss of optokinebc nystagmus, arid
ataxia. The headache is a prominent feature, and is due to obstructive
hydrocephalus. Some pineal tumors are g arm inomas and secrete HOG.
which can cause precocious puberty in prepubertal males.
(Choice A) Although craniopharyngioma can cause a headache, rt cannot
explain the other signs and symptoms of the patient.
Craniopharyogiomas are classically associated with diabetes insipidus
and a deficiency of one or more anterior pituitary hormones.

(C h 0 i ce 13) Frontal lobe tumors can be silent. When symptomatic,
these tumors present with headaches, local neurological deficits, or
seizures. Loss of inhibition from the frontal lithe can cause the
release of primitive reflexes such as glabellar tap, grasp and
palmomental reflexes. Frontal lobe tumors can be associated with
Foster Kennedy syndrome (optic atrophy on the side of the tumor and
papilledema on the c cintralateral side)

{Choicer C) A pituitary tumor with sup ra sell ar extension can cause
bite m poral he m ianopsia Patients with pituitary tumors can present
with hormonal dysfunction, depending on the secretory state and size
of the

tumor

(Choice E) Eye findings in multiple sclerosis are due to involvement
of the medial longitudinal fasciculus, which leads to internuclear
ophthaimoplegia. Bilateral or unilateral optic neuritis can also
occur.

Educational Dlajective:

Pineal tumor class 'calif presents rah Parinaud's syndrome, which is
characterized by the loss of pupillary reaction, vertical gaze
paralysis, the loss of autokinetic nystagmus, and ataxia. Some pineal
tumors are g errn inomas that can secrete HCB.
Q  24

A 70 ­year­old Caucasian man comes to the physician's office for a
routine follow­up visit. He has diabetes mellitus type 2, hypertension
and peptic ulcer disease He reports Mat he had a recent transient
episode of left hand numbness and weakness without any loss of
consciousness. At that time, he did not go to the emergency
departrnent because his symptoms lasted for only fifteen minutes. He
currently has no complaints, and says that he feels 'just great.' His
medications include insulin, Ian soprazole , fosinopril and a m
lodipine. His blood pressure is 150/B0 mmHg and his heart rate is
1341min. Physical examination reveals no abnormalities. CT scan of the
head and color Doppler of the cambric are scheduled. Which of the
hollowing is the most appropriate pharmacotherapy to prevent the
development of a stroke?

A  Aspirin

B   Clopidogrel
C. Ticlopidine
D. Enoxapann
E  Warfarin

Explanation:

diabetes mellitus is currenty considered as an equivalent of coronary
artery disease In this patient, stroke prevention is an important part
of management, especially since he is diabetic, hypertensive, and he
has already had a ­RA­like episode. Since aspinn therapy is
contraindicated in patients with peptic ulcer diseases, clopidogrel
can be used as ark alternative Clopidogrel has been associated with
less gastrointestinal bleeding, and is indicated for those pebents who
have aspirin intolerance or other contraindicabons for its use.
(Choice A) The American Diabetes Association (ADA) recommends aspirin
therapy for the primary prevention of stroke in diabetic individuals
win have other risk factors for cardiovascular disease; however, as
mentioned above, aspirin is contraindicated in this patient because he
has peptic ulcer disease.

(Choice C) Ticlopidine is also effecti•.e for the secondary prevention
of stroke. However. this medication is associated with significant
side effects such as rash. diarrhea, neutropenia and thrombocytopenia;
it requires complete blood count monitonng every 2 weeks for the first
3 months These adverse effects, as well as its high cost, make this
medication an unsuitable a tternabve to aspinn for stroke prevention.

(Choler D) Lew molecular weight hepann (enoxaparin) is not effective
for stroke prevention Furthermore, these are expensive drugs that need
to be administered by the parente ra I route.

(Choice E) Warfarin is not superior to aspirin for stroke prevention
Two large clinical trials have shown that patients who took warfarin
for stroke prevention had more episodes of gastrointestinal hemorrhage
than those who received aspirin Furthermore, warfarin Is
contraindicated in patients with peptic ulcer disease.

Educational Objective;

Aspirin is indicated for the secondary prevention of stroke (in
patients who had a previous cardiovascular events) and for the primary
p revert on of stroke in diabetic patients This Orel along ',Nth
warfarin. is contraindicated in patients with peptic ulcer disease
Patients at high risk of gastrointestinal bleeding and those with
aspirin intolerance can be treated with clopidogrel. Ticlopidine is
associated with significant neutroperna and thrombocOopenia.
Q  25

A 16­year­old Caucasian male presents with decreased hearing and
several subcutaneous nodules. His past medical history is
insignificant His family history is significant for a father with
bilateral deafness that was treated with surgery Physical examination
reveals two hypopigmented spots on his back MRI scans of die head with
gadolinium enhancement show bilateral cerebellopontine angle masses.
Which of the following cell types is responsible for these findings?

A  Astrocytes

B Oligodendrocytes

C.Sachwann  cells

D. Microglia

E. Neural cells

Explanation:

Acoustic neuromas observed in patients with neuronbromatosis. as well
as sporadic. unilateral acoustic neuromas, result from the
proliferation of Sr hwann cells (schwannoma s) Sr hwan n cells are
responsible for die rrrielinization of peripheral nervous system
axons, including axons of most cranial nerves.

(Choice B) 01igod endrocytos are responsible for the myelinization of
CMS axons. including axons of optic nerves.

(Choice A) Astrocytes are sup bort.Ne cells of die CMS. They grre rise
to the majority of glial tumors. (Choice D) mirroglia are composed of
phagosyis cells .

(Choice E) in spite of the name 'neuroma,' acoustic neuromas do not
arise from neural cells

Educational Objective:

Acoustic neuromas result rrom the proliferation of Schwa on cells (schwannomas).

Q 26

A 62­yeari old Caucasian man comes to the physician because of
progressive weight loss and right lower extremity weakness for the
last two months On further questioning, he tells you that he has been
having frequent leg cramps in the right leg These cramps are most
prominent in the morning, after he wakes up He also has difficulty
with swallowing and chewing He denies any problems wth bowel or
bladder function 1­hs other medical problems include hypertension and
hypercholesterolemia. He flans not use tobacco. His family history is
not significant. His vital signs are wrthin normal limits. The
physical examination reveals weakness in his right lower leg. atrophy
and fasciculation of the thigh and calf muscles, a hyperactive knee
jerk, anti tongue fasciculations. The rest of his neurological
examination, including the sensor examination, is normal

Which otthe following is the most likely diagnosis?

A BinsiNanger's disease

B. Vascular dementia

C. Multiple sclerosis
D. Amyotrophic  lateral sclerosis

E Brain stem gliome

Explanation:

Amyotrophic lateral sclerosis (,ALS) is the most common form of
progressive motor neuron disease. It is a relende sshf progressive
disorder that involves both me lower motor neurons (consisting of
anterior horn cells in the spinal cord and brainstern neurons
innervating the bulbar muscles) and upper or corticospi nal motor
neurons. At its onset. ALS may invoPe selective loss of function of
only upper or lower motor neurons, but it ultimate hy causes
progressive loss of both upper and lower motor neurons.

The initial sign of the disease with lower motor neuron involvement is
an insidiously developing asymmetric weakness. us ually first evident
distally in one of the limbs. Patients may disclose a history of
recent development of cramping with volitional movements Mattypical y
occur in the early morning hours (e.g while stretching in bed)
Weakness is associated with progressive wasting, atrophy of muscles,
and spontaneous twitching or (ass iculato ns of motor units.
Involvement of the bulbar muscles leads to difficulty with chewing
arid swallowing, as well as fasciculations of the lace and tongue.

ALS with prominent corticospinal involvement is characterized by hype
ra [Wily of muscle­Stretch reflexes (tendon Jerks) and frequent
spastic resistance to passive movements of the affected limbs. Ocular
motility, sensory, bowel, bladder, and cognitive functions me
preserved, even with advanced disease.

[Chaise B) Vascular dementia is characterized 4y the presence of
behavioral disturbances and cognitive deficits associated with
clinical or radiographic evidence of a stroke. The decline in the
level of cognition is relatively abrupt and progresses in a stepwise
fashion.

(Choice A] Binswanger's disease is a type of vascular dementia that
involves white matter infarcts Patients with this disease usually
present wlh wary. ­agitation, and bilateral cortiCOSpinal or bulbar
signs.

{Choice C) Multiple sclerosis is usually seen in younger females with
two or more clinically distinct episodes of CN,5 dysfunction Patients
typically have sensory, visual. or bladder and bowel dysfunction

(Choice E) Brain stem tumors may compress the Cervcal cord and produce
weakness, tasciEulations in the upper limbs, and spasb E ity in the
legs. The presentation may closely resemble ALS: however, absence of
pain or sensory changes and normal bowel and bladder function in the
patent favor ALS.

Educational Objective:

ALS snsuld OE suspected in patients who present with progressive
weakness accompanied by both upper and lower motor neuron deficits.
Ocular motility. sensory, bowel, bladder, and cognitive functions are
preserved, even with advanced disease.

Q  27

Which of­the following treatment has been shown to he beneficial in
these pahenim7
A  Riluzole

B. CortiCosteroi ds r

C. IV immuno globulins ri

D. Plasrnapheresis C

E Surgery

Explanation:

Riluzole is a glutamate inhibitor that is currently approved for the
management of arryotrophic lateral

s clerosi s. Alta ugh it cannot arrest the underlying pathological
process, it may prolong survival and delay the need tar a tracheosto
roy.

(C h o Ice  B C, and D) Arrnictrophic lateral sclerosis is a
heurodegenerative disease. Agents such as corticosteroids. inbavenous
immunog lobul ins, and c­yc lophosphami de, have no role in its
management. These agents may be useful in immunologically­mediated
neurological diseases such as multiple sclerosis and GiJillain­Barre
syndrome (Cora costeroi ds are used to treat acute exacerbations of
multiple ­sclerosis. while plaamapheresia and intravenous
immunuglobulins are We main treatment modalities or Guillain­Barre
syndrome.)

Educational Objective:

RiluZole is a glutamate inhibitor mat is currently approved for the
management of amyotrophic lateral sclerosis.
Q  28

A 45­ear­old Caucasian man comes to see you in the office. He is
accompanied by his wife. They have noticed that he has shakiness of
both hands for the past six months The shakiness is limited onlyto his
hands and has not progressed over time. It becomes much more
pronounced when he tries to pick up things or attempts to eat or dnnk.
On physical examination, you notice a constant tremor in both his
hands, which becomes much more pronounced when his arms are
outstretched. There is also a very subtle head tremor present. There
is no rigidity or slowing of voluntary movements in the upper or lower
extremities. His father had a similar problem in his 51:I5 or 60s. but
he is not sure what treatment he recawed at that time. His wife is
extremely anxious and wants to know his prognosis

Which of the following is the most appropriate response?

A Most of the patients wth this disease have a normal life expectancy

B. The symptoms will usually resolve over the need 5­10 years_

C. The disease will invariably cause significant disability in the
next 3­6 years.

D. Meditations can completely resolve all the symptoms of this disease.

Explanation:

The oali9rit in the above vignette is most likely suffering from a
familial tremor or benign essential tremor. It is the most common
cause of postural tremor, and its incidence usually increases with
age. The familial cases usually have an a utosomal dominant
inheritance pattem and presents at a relatively younger age.
The tremor in patients with familial disease is usually present in the
distal upper extremities and becomes much more pronounced with
outstretching of the arm. It also increases at the end of an activity
or movement All patients with essential tremor typically do not have
any other neurological signs. The absence of a resting tremor (which
worsens at rest but irnprctoes with voluntary' activity), rigidity
(increased resistance to passive movements), bradykinesia (slowing of
voluntary movements), or gait difficulty differentiates this condition
from an early onset of Parkinson's disease. The presence of a positive
family history and a head tremor is also more suggestive of an
essential tremor

enign essential br familial tremors usually do not cause any
significant disability, and the patient can expect to have a normal
life expectancy. There are no associated neurological symptoms in
patients with benign essential tremors.

(Choice B) The tremor in patients with benign essential or familial
tremors usually worsens with time and can cause difficulty in the
performance of fine motor tesks.

(Choice. C) The tremor can remain stable for years without arty
significant deterioration. It can cause significant disability in
sorne, but not all the patients

(Choice D) Medications. especially beta­blockers. only have a limited
effect on symptom control. They do ..iutLause a complete resolution of
the symptoms_

Educational Objective:

Benign essential tremors or familial tremors generally do not cause
significant disabilities or neurological problems in patents.
Q  29

The patient works as a carpenter and f eels that he is unable to
perform Me fine meter tasks due to the tremor. Which of the follovong
is the most appropriate next step in the management of this patient?

A Do nothing atthis point.

B. Refer him for physical therapy.

C. Start the patient on propranolol

D. Start the patient on levodopa.carbidopa combination.

E. Start the patient on pramipexole.

The patient is suffering from a familial or benign essential tremor
Treatment for familial or benign essential tremor is not necessary in
all patients and should be delayed as long as possible due to the need
for indefinite treatment. The patient, however, has a significant
disability due to the tremorthat is affecting his abilityto perform
fine motor tasks with his hands. Beta­blockers are the most common
first­line agents used in the treatment of benign essential harriers.
Several studies have evaluated the efficacy of propranolol in reducing
tremors and the disability associated with the tremor. In patients
with a history of ro nchosp am, diabetes mellitus. heart block or
other contraindications to beta­blockers, selective beta­1 bloc kers
such as metaprolol or atenolol can be used

(Choices p and B) The patient has symptoms that are affecting his work
as a carpenter. lie should be treated with ph a rmacotherapy at this
point. Physical therapy does not improve the tremor in this condition
orb it not recommended.
(Choices Ci and E) The patient does not have any signs or symptoms of
Parkinson's disease; therefore, a carbidopa­Jevodopa combination or
pramipexole is not indicated in this patient.

Educational Objective:

Beta­blockers are the first­line agents for symptomatic treatment of
patients with benign essential or familial tremor.

Q 30

A 78­year­old Caucasian man is brought to the office by his daughter.
who states Sat he has become forgetful For the past year, his
responses have become slow, and he has been haying increasing
difficulties in caring for himself. He was recently seen by another
physician. who suspected a "certain type Of anemia." He denies any un
nary or respiratory problems. He has hypertension and coronary artery
disease. He takes lisinopril, metoprolo I and aspirin. He lives with
his daughter and son­in­law. He does not use tobacco, alcohol or
illicit drugs. Exarninabon shows an elderly man who is disoriented in
time and space. The mini mental state exam (hiMSE) score is 22 points.
He communicates poorly. He speaks softly and slowly. and seems to
think too much of his answers. His gait is slow and cautious, as if he
were expecting to fall. Laboratory test results show the following'

Which of the following will be the most appropriate next step in the
management of this patient?
Explanation:

This patient presents with dementia, as suggested by his poor
performance in the MMSE Test (A score of less than 24 points is 62%
specific for dementia or delinum in Caucasians), as well as his
history and physical findings. Although mast cases of dementia are due
to Alzheimer's disease, approximately, 2094 of patients have a
potentially reversible cause, such as bryp city oiclism, depression,
etc. The American Academy of Neurology OAT recommends the following
tests in the routine evaluation for patients with dementia' CBC,
glucose, depression screening. serum electrolytes, BUN, rreatinine,
serum B12 levels, and liver function tests In Ow case. the presence of
mocrocytosis, hyponatremia, hradypsyrhe (slowed thought process),
Cess), and slowed speech can be explained by hypothyroidism;
therefore, the most app no pnate next step in the management of this
patient is to determine the levels of 7SI­1.

h oloee A and C) The AAN recommends a head CT scan or MRI for all
patients with dementia, especially if the patient presents %Path motor
deficits or urinary problems. which indicate the possibility of
vascular disease. subdural hematoma or normal pressure hydrocephalus.
A "cautious gait" is normally Found in some elderly patients who are
extreme ty, afraid of falling. The gait may be a marker of
asteoarthritis, but not of any kind of d e F enti a At this point, a
CT scan or MRI of the head is not the hest next step in the patient's
management.

(Choice B) Screening far syphilis in a patient with dementia is
recommended only if the patient has some specific risk factor or
evidence of a prior syphilitic infection, or if the patient resides in
one of the few areas in the U.S. with high numbers of syphilis cases.

(Choice E) Depression is probablythe most cornrne n cause of
reversible dementia; however, it will not explain the macrocytosis or
hyponatrernia in this patient A psychiatry referral is not indicated
in this case.
Educational C ectio e:

Dementia has multiple etiologies. Approximately 60 to GO% of the cases
are due to Alxheimers disease, and 2096 are caused by potenhaiiy
revers dale conditions. The .AAN recommends ommends routine testing
for Vitamin 812 deficiency and hypothyroidism, as well as a head CT or
MRI_ however, the chronological order of these tests should depend on
each patient's clinical manrfestations. Elderly patients with
macrocytosis. hyponatremia, slowed speech and thought processes, and a
delayed relaxation phase of the ankle reflex. are mast likely to have
hypothyroidism as die etiology of their dementia

Q  31

A 65­year­old Caucasian male is brought to your office ay his daughter
because of decreased activity and impaired sleep over the past month
He was diagnosed with Parkinson's disease two years ago According to
his daughter, he displays very few emotions. He wakes up early in the
morning, which is very unusual tar him. He is Compliant 'Nth his
current levadopartarbidopa treatment. On physical examination,
mask­like facies. hand tremors. muscle hype to nus. and bradykinesia
are noted. The patient avoids eye contact and does not want to answer
your questions. Which of the following is the best next step in the
management of this patient?

Decrease the dose of levodoparcatbidopa it­
Add amantadine
C. Switch to selegiline

D. Add a SSRI antidepressant

C E. Consider atypical neuroleptics
Explanation:

Depression affects up to SO% of patients with Parkinson's disease It
can be easily overlooked because some features may he attributed to
the progression of Parkinson's disease. It has a sign rfica nt impact
on disability. and should be highly suspected in patients who present
kth any of the signs of depression leg., early morning awakenings.
poor eye contact. etc). Uncertain cases are usually managed with a
trial administration of SSRI antidepressants before changes in the
medications for Parkinson's disease (Choices

B and C) are considered.

(Choice E) Atypical neuroleptics can be used to treat hallucinations
in patients with Parkinson's disease

Educational Objective:

Identifying underlying depression in patients worth Parlonsonism is
often difficult. An empinC trial of SSRI antidepressants could be
helpful before any changes in the medications for Parkinson's disease
are considered.

Q  32

A 71year­old Caucasian male presents to clinic complaining of 'some
very disturbing whirling attacks." He says that on three occasions in
this past week, he suddenly began to feel dizxy, nauseated, and found
himself unable to walk or speak properly Du nng these episodes he also
noticed a tingling sensation in his lips and that he had double
vision. The symptoms resolved gradually after lasting approximately
eight to ten minutes. The episodes happened at different hmes of day,
both while at rest and during actmty. His medical history is
Significant for diabetes mellitus, chronic obstructive pulmonary
disease, hgertenSion, hypercholesterolemia, and a remote myocardial
infarction. His current medications include mettorrnin, enalapril,
hydrochlonothiazide, sirnvastatin, albuterol, and aspirin. He has a
fifty­pack­ear smoking history end drinks 1­2 beers per night He
denies having ever used recreational drugs His temperature is 36.7C
(98F), blood pressure is 142/88 mm Hg, pulse is 82Irnin, and
respirations are 14/min F undus opic examination shows some
neovascularization of the rehnal surfaces. Tympanic membranes are
translucent and the light reflex is present. Heart sounds are normal.
Auscultation of the chest reveals pitneezing and diffusely decreased
breath sounds. He is barrel­chested. Abdomen is nontender and bowel
sounds are present. Mild peripheral edema is evident There is a loss
of vibratory sensation and altered proprioception and impairment of
pain, light touch, and temperature in a bilateral stacking­glove
distribution Ankle reflexes are decreased His most recent laboratory
evaluation included the follawing:

What is the most likely diagnosis?

A Labyrinthdis

.Benign paroxysmal positional vertigo

C. Vertetrcbasilar insufficiency

D. Panic attacks

E. Cataplexy

Explanation:
Vertebrobasilar insufficiency (Choice C) is a term for reduced blood
flow in the base of the brain. typically secondary to thrombi, or
arterial dissection. The labyrinth and brainstem are commonly
affected, and symptoms may include vertigo, dizziness. dysarthria,
diplopia. and numbness The vertigo often resolves on its oven. but the
condition should not be taken lightly Risk factors for Yertebrobasilar
insufficiency include diabetes mellitus, hypertension,
hypercholesterciemia, arrhythmia, coronary artery disease, circulatory
problems, arid a history of smoking cigarettes.

Labyfinthilis (Choice A) is a mild. often self­limited condition
characterized fey vertigo. tinnitus, nausea. and a loss of balance The
disorder often follows a Nral illness (eg, influenza) Lakyydnthitis
may also be caused by inures, bacterial infection, allergies. benign
tumors, and certain medications

Benign paroxysmal positional vertigo (Choice Bry is en abnormal
feeling of motion triggered by certain provocative positions. The
condition is most often attributed to the presence of calcium debris
within the postonor semicircular canal Nystagmus is commonly seen

Panic attacks (Choice D) are discrete periods of intense fear or
discomfort. Symptoms may include palpitations. sweating, trembling.
shortness of breath, a choking sensation, chest pain, nausea.
dizziness. paresthesias, and a fear of dying or losing control.

Cataplexy (Choice E) is a sudden. temporary loss of muscle tone that
can result in collapse. It is often caused by intense emotions,
including laughter.

Educadonal Objective:

Vertebroba5ilar insufficiency is typically secondary to emboli,
thrombi, or arterial dissection. The labyrinth and brainstem are
commonly affected. and symptoms may include vertigo. dizziness.
dysarthria, diplopia. and numbness.

Q  33

A 55­year­old Caucasian man is brought to the primary care physician
by his Me They live in a very rural area and as a result do not visit
the doctor frequently, but the wife made today's appointment because
'something is just not nght" %Nth her husband. She says, 'His behavior
has gradually changed over the past few years, and he is now different
in that he is very Irritable and moody, keeps repeating certain words
or phrases that I say, and sometimes wanders around aimlessly outside
if 1 am not careful.' She also mentions that he has been licking his
lips Frequently and exhibiting some hyper­oral behaviors Of even more
concern to her is the fact that sometimes. he is silent altogether for
days on end. and he now appears to be experiencing memory loss. Upon
specific questioning, the patient's abilityto plan ahead and set goals
appears impaired He denies that he is any different An MRI of the
train reveals marked symmetric atrophy of the frontal and temporal
lobes. The remainder of the cerebral cortex, cerebellum, and hrainsto
rn appear normal. What is the mast likely diagnosis?

Huntington disease
Lewy body dementia
Pick’s disease
Vascular dementia
Wemicke encephalopathy

Explanation;

Pick's disease is a slowly in re oressive frontal lobe dementia
characterized by speech abnormalities (e.g.. logorrhea, echol a lia.
aphasia. mutism), impaired executive functioning (e g., initiation.
goal setting, and planning). irritable mood. hyper­oral behavior, and
disinhi Who n Cognitive Function is largely normal. Neurornaging
studies reveal the classic finding of prominent symmetric atrophy of
the frontal andlor temporal lobes. The condition may present in a
fashion similar to Alzheimer disease, and the diagnosis is sometimes
not confirmed until the autopsy. Microscopic findings include gliosis,
neuronal loss, and swollen neurons first may contain Pick bodies,
which are silver­staining cytoplasmic inclusions.

(Choice A) Huntington disease is a slowly progressive degenerative
brain disorder Characterized by chorea arid behavioral disturbances.
The chorea usually includes frequent. abrupt jerks of the limbs or
trunk. Grimacing and grunting may occur, and the gait is disjointed
and choreic. Memory is typically preserved until later stages of the
disease, but executive functioning. attention. and judgment may be
impacted early on. Depression and irritability are common

(Choice El) Lewy body dementia is the second most common form of
neurodegenerative dementia after Alzheimer disease. It is a gradually
progressive dementia associated with varying cognitive function and
alertness. persistent visual hallucinations. and some motor features
of parkinsonism (e.g.. rigidity. intention tremor) Falls are common
Microscopic findings in the brainstem and cortex include the presence
of Lew bodies, vuhich are cytoplasmic inclusions.

(Choice D) Vascular dementia is generally classified as multi•infarct
dementia or diffuse white matter dementia The more common of the two.
multi­infarct dementia occurs in patients who have experienced several
strokes that cause chronic cognitive deficits The strokes may be small
or large and typically inyoive multiple regions of the brain. Sudden
neurologic deterioration is a classic complaint, and patients
frequently have a medical history of atherosclerosis, diabetes
mellitus, or hypertension.

(Choice E) Common in alcoholic pad ents with chronic thiamine
deficiencies. Wernicke encephalopathy is characterized by mental
impairment, ophthalmoplegia, horizontal nystagmus, and cerebellar
ntaAa The addib on of memory lass and confabulator/ psychosis define
Wernicke­Korsakoff syndrome,

Eduaational ObJectIve:

Pick's disease is a slowy progressive frontal lobe dementia
characterized by speech abnormalities. impaired executive functioning,
ratable mood, hyper­oral behamor, and disinhibition. Neurairnaging
studies reveal the classic finding of prominent syrnmetn C atrophy of
the frontal a notior temporal lobes.

Q  34

A 73­year­old Mow is brought by her daughter for an examination. She
lives alone in an apartment and continues to drive. Her daughter
reports that her mother began heAng a gradually worsening difficulty
Wth her memory two years ago. She is able to perform most of the
acevines of daily living and cooks for herself, but she has left
burners an for no reason. She has also called her daughter on a number
of occasions from a pay telephone while driving in the local
community, stating that she was lost. Two days ago, she withdrew her
entire MOM savings account. arguing with the bank teller that she
needed the entire amount in $1.1:10 bills Her daughter also remarks
that her personality has become more agitated and aggressive. Her past
medical history and review of systems reveal hypertension, which is
being treated with an ACE inhibitor. Her family history is negative
for dementia On physical examination, she is alert, cooperative,
norrnotensive, and is not depressed Her Mini­Mental State Examination
(MrielSE) scare is 22. She recalled 1 al 3 objects after 5 minutes.
She is unable to draw a clack and states, 'Who pays attention to the
date? I'm retired' She is disoriented to time and place. Her
neurologic examination is otherwise normal, with no lateratizing signs
and no extrapyramida I symptoms. Her gait is normal. What is the next
best step in the management of this patient?

A.    Arrange for the patient to he admitted to a chronic care
facility and placate the daughter.

B   Prescribe donepezi I (Aricept) for the patient and advice the
daughter to drive for her mother for 3 weeks.

C. Order a complete blood count. serum glucose, electrohytes, calcium.
cre atini ne. and TSH..

D. Prescribe haloperido I (Haldol) for the patient arid diazepam for
the daughter

E. Refer the patient for immediate consultation with a geriatrician

Explanation:

This patient exhibits a moderate, gradual onset, progressive, preseni
le, cortical dementia. A subcortical dementia is unlikety without gait
or other motor disturbances. Stahstica Ily. the most likely cause is
4izheimers disease (AD). however, AD is a diagnosis of exclusion. A
dementia worli,i,up locking for reversible causes must be completed
before a clinical diagnosis of AD can be made. Since this patient has
a history of hypertension, one might consider multi­infarct dementia
In this patient, the absence of a step­vase decline in cognitive
function and of any neurologic signs such as hemiplegia, pseudo bulba
r palsy with pathologic laughing and Crying, or other extrapwarnidal
dysfunction argues against a mechanism of multiple sequential small
strokes. A brain CT or MRI scan I ooldng for la C unar infarcts is
probably unnecessary. especially since the patient's blood pressure is
apparently under control. Among the many other possible causes of
dementia. it is most cost effective to rule out die more common
reversible causes. namely a nutritional or other anemia,
hyperglycemia, an electrolyte disturbance, hyper­ or hypo­calcemie,
renal fa/lore, and frypothryroid ism; hence, the laboratory tests
listed in this answer should be obtained before arty final diagnosis
and further management. Other tests, including a head CT scan, may tie
done rf there is a specific clinical indication (e.g., n13 and folate
If rnacrocytosis is present).

Optional: The MMSE score of 22 is also ronsistentwith a mild to
moderate dementia a See below. Table 1. Mini ­ Mental Status
Examination

Orientation Maximum
Score'
Patient
Score'

What is the (year) (season) (date) (day) (month)?
Where are we, (state) (country) (town) (hospital) (floor)
5 I
5

Registration

Name 3 objects:
Examiner is to say each, then ask the patient all 3 after saying

them Give I point for each correct answer Then repeat them
3
(Choice A) This step WO uld be premature until the dementia work­up
and neuropsychiatric evaluation is completed. it is inappropriate to
arrange institutional chronic care for a confused elderly patientwith
Impaired memory and judgement until reversible causes have either been
treated or ruled out.

(Choice B) This choice might be an appropriate next step if the
dementia work­up was negative, and if a clinical diagnosis of
Alzheimer's disease (AD) had already been made Acetylcholinesterase
inhibitors such as tacnne (Cogn ex), don epezil )Aricept), nvasdgmine
(Exe Ion), and galantarnine (galanthamine, Peminyl) are commonly
prescribed. Complete onset of the therapeutic effect of these drugs
may not be obtained for a few weeks after initiation.

(Choice D) Although this option may sound humorous. there are some pad
ents with en established diagnosis of Alzheimer's disease tor whom
such treatment vvould be applicable A neuroleptic drug (major
h­anquilizer) may be prescribed for episodes of aberrant andfor s DC
ially d isruphve behavior that can occur. During such episodes_ family
members may also require support and a sedative.

{Choice E) Although such a referral might be considered, the physician
performing the patient's intial evaluation should at least commence a
dementia work­up More appropriately, as part of the dementia work­up,
the primary physician should also arrange for a neuropsychiatric
evaluation of an elderly patient nresenhng 'oath increasing confusion,
memos} impairment, and a decreasing ability to look after himiherself.

Educational Objective

Alzheirner s disease is a diagnosis of exclusion. It is inappropn ate
to arrange institutional chronic care for or to treat )vi4th an an
acetylchalinesterase inhibitor) a confused elderly patient with
impaired memory and judgement unhl a dementia work­up has been
completed to rule out reversible causes.
Q   35

You are the current 'on call physician" for your group, when you
receive a call regarding one of your nursing home patients. The nurse
reports that one patient has been acting strangely for the last few
hours and is agitated at trite s. This patent is an B2­year­old female
who has been in the nursing home for the last two years, and has
always been very pleasant. You know her very well from your recent
nursing home visit. She has a past medical history of hypertension.
diabetes mellitus. osteoporosis. and Alzheimer's dementia. Her daily
medications are amlodipine, metformin. calcium • vitamin D, and
donepezil. You are concerned that something is not right with the
patient.

Which of the following is the most appropriate rieA step in the
management of this patient?

A. Transfer the patient to a nearby hospital

B. Give a dose of haloperidol

C. Ask the nurse to check vital signs and pulse oximetry

D.Give diazepam for sedation

E Ask the nurse to use physical restraints as necessary to
preventfalls andl injury

Explan ati o n:
This patient is sufrenng from an acute confusional state, also known
as delirium. Delirium is extremely common in old nursing home
patients. It manifests as a change in the level of awareness, easy
distractibility, cognitive and perceptual disturbances including
memory loss, disorientation, and even agitation. It typically develops
over hours to days, and thus acute onset distinguishes it from
dementia.

Dementia is a mull:doctorial disorder The factors that increase the
risk of delirium are subdivided into those that increase the
susceptibility to delirium and those that precipitate delirium. The
most common factors that increase the patient's susceptibility to
delirium include advanced age and an underlying brain disorder (i.e.,
dementia, Parkinson's disease, and history of prior stroke). The
factors that precipitate delirium include multiple medication use
(polypharmacy), infections (i.e., urinary tract infections and
pneumonia), fluid and electrolyte disturbances (dehydration. hypo
natremia or hrypematrernia), malnutrition. immobility (including the
use of restraints), use of bladder catheters, and a wide variety of
other medical conditions In fact, delirium may be the only presenting
complaint in an older demented patent suffenng from an acute medical
illness; therefore, checkng this patient's vital signs should be die
first step to rule out early infections and fluid and electrolyte
disturbances.

(Choice A) Transferring this patient to a hospital may be necessary
for further evaluation of the cause at the delirium; however, this
should not be the first step in the management of this patient

(Choices E and D) Misuse of antpsychotics and ben z odiazepines is
very common in managing behavioral problems in demented nursing home
patients It leads to frequent adverse effects and worsening of doe
cognitive status; therefore, it is important to Rile out any medical,
errn nrnental, or psychosocial causes before prescribing any
psychotropic medications.

(Choice E) As explained above. the use of physical restraints can
precipitate and even worsen the delirium, arid should therefore be
avoided

Educational objecdve:

Delirium can tie the only finding of an acute medical condition in
older demented individuals. Use of psyc hotropic medications n a first
line therapy for behavioral problems is not recommended

Q   36

The nurse calls you back in tvvenv minutes. She states that she gave a
dose of pm (as needed) medication which was ordered by a nursing
physician before. She arranged an ambulance In transfer the patient to
a nearby hospital. Her temperature is 36.7 C (P F), blood pressure is
116(113 mmHg, pulse rate is 76/min, and respiratory rate is 113/min.
She is sbll agitated, and she is now disoriented to place and person.
She gets easily distracted. arid she is having visual and auditory
hallucinations In the hospital. her physical examination is
unremarkable What is the most appropriate next step in the management
of her condition?

A Give he loperidol to calm die patient

B Order a stat psychiatry consult

C. Order  urine analysis and complete metabolic panel

D. Order a head CT scan

E Order physical restraints to prevent injury to patient and nursing staff
Explanation:

This patient is still exhibiting the classic signs and symptoms of
delirium Cognitive and perceptual disturbances including
hallucinations are very common in delirious patients.

As described above. virtually any acute medical condition can
precipitate delirium in an older demented patient It is important to
perform a complete physical examination and obtain basic laboratory
studies to identify or rule out the potential causes of delirium,
especial y the reversible causes, which include' dehydration,
hyperhypernatremi a infections. hypoglycemia, hypercalcemia, uremia,
and liver failure.

(Choices A and B) The use of psychotropic medications to calm this
patient is not recommended at this time It is prudent to rule out the
reversible medical causes of delirium before prescribing psychotropic
medications or obtaining a psychiatric consultation.

(Choice D) Pere is no history of head trauma or any focal neurological
signs and symptoms: therefore. a head CT scan is not indicated at this
point.

(Choice­ E) Physical restraints should be used only as a last resort
in an agitated and delirious patient. They frequently increase
confusion and agitation. Constant observation, preferably by someone
familiar to the patient, is more helpful in such situations.

Educational obj•ctiya:

Focused history, physical examination and basic laboratory work­up
should be performed early in die initial evaluation of delirium in any
patient.
Q  37

A CT scan of the patient's head is normal. India/ lab studies reveal
the presence of a urinary tract infection with mild prere nal
a2oternia. You start die patient on empiric antibiotic therapy and
intravenous fluids, however. the patient's mental status remains
unchanged. You provide a bedside sitter for the patient for constant
supenasion The sitter reports that the patient is very combative now.
She has been trying to get out of bed and has twice attempted to pull
her IV line out She resisted the physical restraints that the nurse
was trying to apply. Frequent orientation and reassurance has not
improved her behavior. Which of the Fallowing the most appropriate
next step in the management of this patient?

A Give low dose haloperidol

B. Give low dose lorazepam

C. Order physical restraints to prevent injury anyway

D  Provide another sitter for the patient

E. Perla rm lumbar puncture

Explanation:

The patient is exhilt Ong behavior that could harm herself and the
healthcare staff. Initial supportive measures such as reassurance,
orienting her to the environment, and haying constant supervision has
not been successful in managing her disruptive behavior. Low­dose halo
peridoi may be useful in controlling her symptoms, preventing harm.
and allowing kw the safe continuation of her treatment.Andpsychatics
are associated vital a variety of side effects These include sedation.
orthostatic hypotension. anticholinergic side effects, terrific
dyskinesia, and extrapyramidal symptarris Thus, the patient should be
started at lower doses and the dose is then otrated upwards to achieve
the desired effects

(Choice B) Benzo Clamp ines have a more rapid onset of action. but
these can worsen confusion and sedation They can be used as an adjunct
to antipsychotics tO reduce extrapyramidal side effects.

(Choice C) Physical restraints should be only used as a last resort in
a combative patient. They frequently lead to worsening of confusion.
They are used when medications fail to act, and the patient remains a
significant threat to his/her and the healthcare staffs safety

(Choice D) Sitters are helpful in providing onentahon and constant
supervision for a confused patient. Once a patient becomes combative.
other modalities including chemical and physical restraints are
frequently necessary

(Choler E) Performing a lumbar puncture is only indicated if the
patient has unexplained fever, altered mental status, and nuchal
rigrdity Change in mental status alone rs not an indication for lumbar
puncture.

Educational objective:

Physical and chemical restraints should be used Judiciously in a
combative and disruptive patient These help prevent injuries and
falls, and allow for the continuation of medical therapy. Always give
halal] endol (antipsycholics) before you apply restraints.
Q   38

A 72­year­old Caucasian woman is brought to your office by her son
because of her impaired memory and cognition. Her son was out of the
count!), for two years while working for an international
organization. He recently returned, and found his mother disheveled
and poorly oriented. Her gait is shuffling. and her movements and
speech are slow. Her neighbors say that the decline in her functioning
progressed slowly aver the last year. Physical examination reveals no
localizing neurological signs.

Which of the following is the most likely cause of cogniove impairment
in this patient?

A. Diffuse cortical atrophy

B Recurrent cerebrovascular events

C. Subcortical  dementia

D. Frontal lobe lesion

E. Pseudo­dementia

Explanation:

Prominent extrapyramidal symptoms in this patient suggest subconical
dementia as the cause of her cognitive decline, shuffling gait and
slowing of movements and speech These symptoms descd be The
'SUL:Cannel pattern' of dementia, and distinguishes it from the
'cortical pattern'• with minimal eydapyramidal symptoms (e g.,
Ahheirner's disease) Subcorbcal dementia encompasses several clinical
entities. and includes Parkinson's disease d emenh a. progresstre
supranuclear palsy, etc. Dementia with Lem bodies is the most comma n
type that is associated with Parkinson's disease. It is estimated that
about 30% of patients with Parkinson's disease develop dementia during
their course of illness.

(Choice 8) Vascular dementia is characterized by a stepwise
deterioration of cognitive function, and is typically accompanied by
focal neurological signs.

(Choice D) Frontal lobe syndromes g , Picks disease) present widi
behavioral changes (e g disinhibited E hoe or) and various language
impairments (e.g logorrhea, echo lali a, palilalia).

(Choke E) Pseudo dementia is a form of reversible dementia associated
with depression. Extrapyramidal symptoms we not present.

Educational Cbjectivir

Extra pyramid al symptoms are useful in describing the 'subcortital
pattern' of dementia to distinguish it from the 'cortical pattern'.
which is characteristic for Alzheimer's disease.

Q39

Two weeks after the initial \1st to your office, the patent is brought
to the emergency department (ED) by her son due to agitation and
restlessness She says, "I see people in colorful clothes around me,
and I try to speak with them, but they do not respond. These people
are sent by God to take me we, them.' The son says that these symptoms
started two days ago, during the night, and progressed slowly. She
refuses to eat arid go to the bathroom. Which of the following is the
most likely cause of these symptoms?
A. Acute psychosis

B A new episode of stroke

C.Levodopa  treatment

D. SSIR, treatment

E. Post­ictal state

Explanation:

This patient has vivid visual hallucinations which are very
characteristic of subcortical dementia associated with Parkinson's
disease. The typical provoking factor of these hallucinations is the
initiator' of levodopa treatment. Levude pa was most trot ably
prescribed after the initial visit to decrease rigidity and
bradykinesia. and improve overall functioning of the patient. The
visual hallucinations, confusion and agitation on levodopa treatment
are highly suggestive or dementia with Lew,/ bodes.

(Choice D)  SSRI treatment has shown to reduce psyc hot C and
behavioral disturbances in patients with dementia, unlike bicyclic
antidepressants, which have prominent anticholinergic properties.

(Choice E) A post­ictal state may cause confusion and lethargy, but is
not accompanied by vivid visual hallu nab ons, and rarely lasts more
than 24 hours.

Você também pode gostar